Die ultra-diffuse Galaxie NGC 1052-DF2; die um sie kreisenden Kugelsternhaufen sind mit mit Zahlen markiert, siehe Text. Bild: [1]

Neulich ging eine Meldung durch die Presse, dass eine Galaxie (fast) ohne Dunkle Materie gefunden worden sei. Während die wesentlichen Schlussfolgerungen anderswo schon gezogen wurden, möchte ich hier ein wenig näher auf die zugrunde liegende Arbeit eingehen und erläutern, was da eigentlich wie gemessen wurde.

 

Worum geht es?

Am 27.03.2018 veröffentlichte eine Gruppe um den an der Yale-Universität lehrenden niederländischen Astronomen Pieter van Dokkum eine Arbeit über die Galaxie NGC 1052-DF2, die sich in der Nähe der elliptischen Galaxie NGC 1052 im Sternbild Walfisch befindet. Sie war vorher schon bekannt aber bisher noch nicht auf ihre Masse hin untersucht worden. NGC 1052-DF2 zählt zu den ultra-diffusen Galaxien (UDGs), die verhältnismäßig groß sind (über 1,5 kpc = 4900 LJ Radius) aber dabei trotzdem sehr lichtschwach (mehr als 24. Größenklasse pro Quadrat-Bogensekunde). NGC 1052-DF2 hat einen Halbmesser von 7700 LJ gemessen bis zu dem Radius, bei dem die Helligkeit auf den Wert 1/e der zentralen Helligkeit abgenommen hat, und hat in diesem Radius im Mittel eine Helligkeit von 24,4 Größenklassen je Quadrat-Bogensekunde. Sie ähnelt damit anderen UDGs. UDGs sind deswegen so lichtschwach, weil sie nur rund 1/1000 der Sterndichte enthalten wie etwa die Milchstraße als typischer Vertreter von Spiralgalaxien. Eine solche Galaxie von Milchstraßengröße hat nur in der Größenordnung von 108 (100 Millionen) Sonnenleuchtkräften, die Milchstraße hingegen ca. 2·1010 (20 Milliarden).

Eine aktuelle Theorie besagt, dass solche Galaxien zunächst wie ganz gewöhnliche Galaxien aus kollabierendem Gas und Dunkler Materie entstehen. Dunkle Materie (s.u.) soll gemäß der heutigen Theorien bei der Galaxienentstehung eine zentrale Rolle spielen, weil sie (im Schnitt) in vierfacher Menge der gewöhnlichen Materie im All vorkommt, ihre Verdichtung die gewöhnliche Materie mit sich zieht und den Entstehungsprozess beschleunigt. Die Galaxien bilden bei der Entstehung in Abhängigkeit von der Menge des verfügbaren Gases eine Zahl von Kugelsternhaufen aus, die deswegen alle ungefähr so alt wie die Galaxie selbst sind. Auch die UDGs bildeten Kugelsternhaufen aus, die auf eine große anfängliche Masse von Gas schließen lassen, verloren jedoch während der Entstehung irgendwie einen Großteil dieses Gases, etwa durch Supernovae. Aufgrund des Mangels an Gas wurde die Sternentstehung früh unterbunden und es entstand nur ein Bruchteil der Sterne im Vergleich zu anderen Galaxien mit ähnlicher Anfangsmasse – es blieb im Wesentlichen nur die Dunkle Materie übrig. UDGs wären demnach fehlgeschlagene Galaxien.

In der Vergangenheit wurden dementsprechend nur UDGs gefunden, die einen sehr hohen Anteil an Dunkler Materie haben, ja teilweise geradezu nur aus Dunkler Materie bestehen. Die von van Dokkum in Kooperation mit Roberto Abraham, University of Toronto, gefundene Galaxie Dragonfly 44 (benannt nach dem Dragonfly Telephoto Array, mit dem sie entdeckt wurde, siehe Bild) enthält beispielsweise 99,9% Dunkle Materie und hat nur 240 Millionen Sonnenleuchtkräfte bei annähernd gleicher Masse wie unsere Milchstraße, 1012 (= 1 Billion) Sonnenmassen.

Das Dragonfly Telephoto Array der University of Toronto, Kanada, mit Team (ganz links: Roberto Abraham, ganz rechts: Pieter van Dokkum). Das Gerät besteht aus einer Batterie von Kameras mit 400-mm-Canon-Teleobjektiven und erinnert an ein Insektenauge – daher der Name (dragonfly = engl. Libelle). Bild: © <a href="https://www.dunlap.utoronto.ca/instrumentation/dragonfly/">University of Toronto</a>, mit freundlicher Genehmigung.

Das Dragonfly Telephoto Array der University of Toronto, Kanada, mit Team (ganz links: Roberto Abraham, ganz rechts: Pieter van Dokkum). Das Gerät besteht aus einer Batterie von Kameras mit 400-mm-Canon-Teleobjektiven und erinnert an ein Insektenauge – daher der Name (dragonfly = engl. Libelle). Bild: © University of Toronto, mit freundlicher Genehmigung.

Und nun hat das Team um Dokkum festgestellt, dass NGC 1052-DF2 so gut wie gar keine Dunkle Materie enthält.

 

Die dunkle Seite der Materie

Dunkle Materie ist bekanntlich das, was Galaxien und Galaxienhaufen offenbar zusammenhält. Misst man die Geschwindigkeiten der Rotation von Spiralgalaxien  oder wie schnell sich Galaxien in Galaxienhaufen umeinander bewegen, dann sind sie viel schneller unterwegs, als es die sichtbare Materie eigentlich zuließe. Wenn man die sichtbare Masse bestimmt, stellt man fest, dass weniger als 20% der Masse zu sehen sind, die nötig wären, um die Objekte bei den beobachteten Geschwindigkeiten zusammen zu halten. Sie sind schneller als mit der Fluchtgeschwindigkeit der sichtbaren Materie unterwegs und sollten eigentlich in alle Richtungen davon fliegen. Daher haben Jan Henrik Oort und Fritz Zwicky bereits in den Jahren 1932 bzw. 1933 vemutet, dass es in der Milchstraße und im Coma-Galaxienhaufen eine nicht sichtbare Materie geben muss, die die fehlenden 80% ausmacht und das Ganze zusammen hält. Sie muss sich dabei so wie ein ideales Gas niedriger Temperatur verhalten – sie stößt mit nichts zusammen, sie bewegt sich langsamer als die Fluchtgeschwindigkeit ihrer eigenen Masse und spürt die Gravitation, und sie sendet keinerlei Licht oder andere elektromagnetische Strahlung aus, weder eigene, noch reflektierte. Bis heute wissen wir nicht, was die Dunkle Materie ist. Nur was sie nicht sein kann:

  • Sterne – würden sich durch ihre Leuchtkraft verraten, sie wären nicht dunkel.
  • Gas – würde von den Sternen zum Leuchten angeregt oder erwärmt und somit messbare Strahlung (Licht, Infrarot, Radiostrahlung) aussenden und wäre also auch nicht dunkel. In der Durchsicht in Galaxienhaufen verrät es sich, indem es Absorptionslinien mit anderer Rotverschiebung im Licht von weiter entfernten Hintergrundobjekten verursacht. Das Gas in der gesamten Milchstraße kann mit Radioteleskopen direkt beobachtet werden. Beides reicht bei weitem nicht aus für die Dunkle Materie.
  • Staub – blockiert das Licht dahinter liegender Sterne und verrät sich in der Durchsicht (wo Sternenlicht noch durchdringt) durch eine Rotverfärbung des Lichts (NICHT Rotverschiebung, es verändern sich keine Wellenlängen, sondern der blaue Teil des Lichts wird stärker vom Staub verschluckt als der rote) und wie Gas durch Radiostrahlung. Auch der Staub reicht nicht, er macht nur einen kleinen Teil der Materie aus und spielt zwischen den Galaxien, wo noch hautpsächlich das ursprüngliche, metallarme Gas des Urknalls herum hängt, keine Rolle.
  • Planeten, Planemos, Braune Zwerge, Neutronensterne, stellare Schwarze Löcher – nach solchen Objekten (Massive Compact Halo Objects – MACHOs) hat man mit statistischen Methoden gesucht, denn sie würden sich durch Graviationslinsenereignisse im Licht entfernterer Sterne verraten. Solche wurden auch beobachtet, aber viel zu wenige.
  • Überhaupt “normale” baryonische Materie – Baryonen sind Teilchen aus 3 Quarks, also insbesondere Protonen und Neutronen in den Atomkernen, die 99,95% der Masse der Materie ausmachen, aus denen alles um uns herum und wir selbst bestehen. Den Baryonenanteil an der Gesamtmasse kann man aus dem Leistungsspektrum der kosmischen Hintergrundstrahlung und der Zusammensetzung des beim Urknall entstandenen Gases abschätzen. Man kommt so nur auf 20% der benötigten Materiedichte – eben die sichtbare, baryonische.
  • Diverse hypothetische “schwach wechselwirkende massive Teilchen” (Weakly Interacting Massive Particles, WIMPs; wimp = engl. “Schwächling” – der Kontrast zu den o.g. MACHOs war durchaus beabsichtigt!). Zahlreiche Experimente wie etwa XENON konnten viele Massen und Größen (“Wirkungsquerschnitt”) solcher Teilchen ausschließen. Es gibt natürlich immer noch einen Bereich besonders kleiner oder massiver Teilchen, der nicht ausgelotet wurde.
  • Auch mutmaßliche supersymmetrische Partnerteilchen der normalen Materie wurden bisher nicht aufgespürt, eine der Aufgaben, die der LHC am CERN lösen sollte. Man hofft aber immer noch darauf, etwa den mutmaßlichen supersymmetrischen Partner des Top-Quarks (das STOP) bald zu finden. Der LHC läuft mit voller Kollisionsenergie und sucht.

Es gibt auch noch andere valide Kandidaten wie das hypothetische Axion oder sterile Neutrinos, eventuell gar während des Urknalls entstandene (“primordiale”) Schwarze Löcher (anderer Masse als diejenigen, die aus Sternen entstanden), aber gefunden hat man, so viel steht jedenfalls fest, noch nichts.

Dennoch muss da irgendetwas sein. Die Dunkle Materie verrät sich durch die Wirkung ihrer Schwerkraft auf Galaxien und Galaxienhaufen. Man kann ihre Schwerkraft mit modernster Computertechnik aus den leicht verzerrten Bildern von Galaxien rekonstruieren und ihre Verteilung in Galaxienhaufen und um Galaxien herum sichtbar machen. Ohne Dunkle Materie ließe sich weder die flache Geometrie des Universums noch die Entstehung  der Strukturen im Universum erklären. Ethan Siegel hat das in einem aktuellen Artikel schön zusammengefasst.

Ich kann an dieser Stelle das Thema “Dunkle Materie” nur anreißen. Manchem mögen die oben genannten Begriffe wie katalanische Dorfnamen vorkommen, zu jedem der Punkte oben könnte ich einen ganzen Artikel schreiben. Werde ich irgendwann auch einmal tun, bis dahin verweise ich auf Florian Freistetters umfassende Blog-Serie über Dunkle Materie.

 

Wie wiegt man eine Galaxie?

Zurück zu van Dokkums Arbeit. Wie kann man überhaupt die Masse einer Galaxie bestimmen? Wenn man auf der Erde etwas fallen lässt, hat es eine bestimmte Fallbeschleunigung. Abgesehen vom Luftwiderstand fallen alle Objekte gleich schnell. Auf dem Mond mit seiner kleineren Schwerkraft fallen Objekte ebenfalls gleich schnell (hier gibt’s auch keinen Ärger mit dem Luftwiderstand), nur deutlich langsamer als bei uns, wie der Astronaut David Scott einst bei der Apollo-15-Mission demonstriert hat. Die Schwerebeschleunigung des Mondes ist an seiner Oberfläche nur 1/6 derjenigen der Erde. Das liegt daran, dass der Mond nur 1/81 der Erdmasse hat und man sich außerdem auf seiner Oberfläche näher am Schwerpunkt des Mondes befindet, denn die Schwerkraft hängt natürlich auch vom Abstand zum Objekt ab. Im Abstand des Erdradius vom Mondmittelpunkt herrscht tatsächlich nur 1/81 der Erdschwerebeschleunigung.

Man kann also aus der Schwerebeschleunigung auf die Masse eines Himmelskörpers schließen. Statt Objekte auf ihn fallen zu lassen, was insbesondere bei fernen Galaxien nicht ganz einfach ist, kann man sich alternativ anschauen, wie schnell vorhandene Objekte um den Himmelskörper kreisen, denn auch eine Umlaufbahn ist “freier Fall” und wird von der Schwerebeschleunigung und damit der Masse des Himmelskörpers beeinflusst: je größer die Masse, desto schneller muss ein Objekt bei gegebenem Abstand um die Masse kreisen, um nicht herunter zu fallen. Aus der Umlaufzeit des Mondes um die Erde kann man beispielsweise die Masse der Erde berechnen (und aus derjenigen der Erde um die Sonne deren Masse, siehe gleichen Link, unten).

Das funktioniert hervorragend bei Objekten, deren Umlauf man wenigstens ein Stück weit beobachten kann wie etwa bei Doppelsternen – so werden Sternmassen bestimmt, ein paar Jahre Wartezeit reichen gemeinhin. Galaxien werden von Kugelsternhaufen umkreist, so auch NGC 1052-DF2. Das Problem ist nur, dass die Umlaufzeit in Jahrmillionen gemessen wird, da kann man buchstäblich lange darauf warten, ein noch so kleines Stückchen vom Orbit zu sehen.

Glücklicherweise liefert einem die Spektroskopie jedoch wenigstens eine Geschwindigkeit sofort, nämlich die radiale Komponente, auf den Beobachter zu oder von ihm weg. Diese alleine für nur ein einzelnes Objekt im Orbit hilft einem zwar nicht wirklich weiter – der Kugelsternhaufen könnte rasend schnell von Nord nach Süd über den Himmel sausen, und wir bemerkten es nicht. Wenn man aber mehrere Objekte betrachten kann, dann kann man mit Statistik an die Sache heran gehen. So kann man beispielsweise bei zufälliger Verteilung und Positionen von Galaxien in einem Galaxienhaufen den Virialsatz über die Verteilung von kinetischer und potenzieller Energie anwenden und aus diesem eine Beziehung zwischen der Streuung (Dispersion) der Radialgeschwindigkeiten und der Massensumme der Galaxien herleiten. Man muss also nur genug Radialgeschwindigkeiten messen und kann daraus die Masse des Haufens bestimmen.

Die Autoren des van-Dokkum-Papers haben genau diese Streuung der Geschwindigkeiten für 10 identifizierte Kugelsternhaufen von NGC 1052-DF2 anhand des Dopplereffekts auf ein Triplett von Kalzium-Spektrallinien gemessen. Sie ermittelten eine Streuung der Radialgeschwindigkeiten von 8,4 km/s und schließen daraus unter Berücksichtigung von Messfehlern auf eine Streuung der realen Raumgeschwindigkeiten von nur 3,52 +5,5/-3,2 km/s, mit 90% Sicherheit aber weniger als 10,5 km/s. Die Autoren verwendeten nicht die oben verlinkte einfachen Virialsatz-Beziehung, sondern eine vom Prinzip her ähnliche Massenabschätzung gemäß einer Arbeit, die speziell den Fall von Satellitengalaxien betrachtet, die um eine große Zentralmasse kreisen (nämlich Satelliten der Milchstraße und großen Andromeda-Galaxie). Mit dieser Methode ermittelten van Dokkum & Co. die Masse für zwei Radien der Kugelhaufenorbits (dem mittleren Abstand der Kugelhaufen zur Galaxie von ca. 10.000 LJ und dem des fernsten Kugelhaufens von knapp 25.000 LJ) und kamen auf 320 bzw. 340 Millionen Sonnenmassen. In der Lokalen Gruppe, d.h. der Gruppe von Galaxien, zu der die Milchstraße und die  Andromedagalaxie gehören, haben nur solche Kugelhaufen so geringe Geschwindigkeitsdispersionen, die Zwerggalaxien von gerade mal 600 Lichtjahren Durchmesser und höchstens 2 bis 3 Millionen Sonnenmassen an leuchtender Materie umkreisen – die aber eben auch eine im Verhältnis große Menge an Dunkler Materie beinhalten, welche in Galaxien weitaus stärker im Vergleich zur sichtbaren Materie konzentriert sein kann, als im mittleren Mix des intergalaktischen Raums.

Spektren der 10 Kugelsternhaufen. Die Geschwindigkeiten cz wurden gemessen, Δv sind die Geschwindigkeitsdifferenzen zur Muttergalaxie (1803 km/s).

Spektren der 10 Kugelsternhaufen. Die Geschwindigkeiten cz wurden gemessen, Δv sind die Geschwindigkeitsdifferenzen zur Muttergalaxie (1803 km/s). Bild: [1]

Theoretisch besteht die Möglichkeit, dass die Kugelsternhaufen von NGC 1052-DF2 nicht gleichmäßig verteilt sind, sondern beispielsweise alle in einer Ebene kreisen, was den Ansatz zur Massenabschätzung konterkarikieren würde – wenn man von oben auf die Ebene blickte, sähe man nur geringe Radialgeschwindigkeiten. Daran haben die Autoren gedacht und die Geschwindigkeiten in Abhängigkeit vom Abstand in vertikaler und horizontaler Richtung zum Massenzentrum aufgetragen. Sie fanden jedoch nicht den für diesen Fall zu erwartenden Trend.

Die Geschwindigkeiten der Kugelhaufen in Abhängigkeit vom Abstand zum Zentrum in zwei Achsen. Es ist kein rechts-links- oder oben-unten-Trend erkennbar, wie er zu erwarten wäre, wenn die Kugelhaufen in einer Ebene kreisen würden. Bild: [1]

Wieviel leuchtet da?

Die Galaxie ist also leicht und lichtschwach, das ist noch nicht sonderlich bemerkenswert. Es fehlt noch die Abschätzung der leuchtenden Masse. Dazu braucht man zunächst die Entfernung. Diese wurde auf zwei unabhängige Weisen bestimmt, einmal durch die kosmologische Rotverschiebung vermindert um die Beschleunigung durch umliegende Galaxienhaufen, 66,5±3,3 MLJ, und einmal aus statistischen Schwankungen der Flächenhelligkeit (surface brightness fluctuations, SBF) durch verschiedene Zahl von Riesensternen pro Pixel, 62±5,5 MLJ – was es nicht alles gibt!

Die leuchtende Masse haben die Autoren nun auf zwei voneinander unabhängige Weisen bestimmt. Eine einfache Methode schätzt die Masse einfach aus der beobachteten Helligkeit ab. Für Kugelsternhaufen (ähnlich alt, ähnlich lange keine Sterne mehr gebildet wie die Galaxie, also ähnliche Sternenpopulation) ist bekannt, dass zwei Sonnenmassen etwa eine Sonnenleuchtkraft haben (M/Lv=2,0).  Mit der ermittelten Entfernung kann man die Leuchtkraft aus der scheinbaren Helligkeit berechnen und kommt auf 110 Millionen Sonnenleuchtkräfte. Macht also mit der obigen simplen Masse-Leuchtkraft-Beziehung für Kugelsternhaufen 220 Millionen Sonnenmassen an leuchtender Materie.

Die andere Methode zur Schätzung der leuchtenden Masse verwendet ein Computermodell, das eine im Abstand von 20 Mpc =  65,2 MLJ modellierte Galaxie mit der gleichen Morphologie, Helligkeit, Farbe, SBF, Metallgehalt und Alter aus einer initialen Massenfunktion (IMF) nach Kroupa (übrigens einem Gegner der Dunklen Materie) erzeugt. Die initiale Massenfunktion sagt aus, wieviele Sterne welcher jeweiligen Masse aus einer kollabierenden Gaswolke entstehen; nach den modellierten 11 Milliarden Jahren Alter sind die massivsten Vertreter dann zu massiven Sternenleichen (Neutronensterne, Schwarze Löcher) geworden und Sterne von Sonnenmasse sind gerade Rote Riesen. Wenn man alles korrekt abstimmt, kommt man auf eine Modellhelligkeit pro Sternenmasse und kann aus der tatsächlich beobachteten Helligkeit auf die Masse zurück schließen. Und auch hier ergaben sich etwa 200 Millionen Sonnenmassen an leuchtender Materie.

 

Und was heißt das jetzt?

340 Millionen Sonnenmassen gravitativer Masse stehen also 200 Millionen Sonnenmassen leuchtender Materie gegenüber. Das ist vergleichsweise wenig für eine UDG – bei Dragonfly 44 waren es 240 Millionen Sonnenleuchtkräfte, also bei entsprechender Masse-Leuchtkraft-Beziehung M/Lv=2,0 knapp 500 Millionen Sonnenmassen im Gegensatz zu 1 Billion Sonnenmassen an gravitativer Masse – ein Verhältnis von 1:2000! Selbst normale Galaxien mit 200 Millionen Sonnenmassen leuchtender Materie haben Halos von Dunkler Materie in der Größenordnung von 60 Milliarden Sonnenmassen, ca. 400-mal mehr als die 150 Millionen Sonnenmassen, die von den Autoren als obere Schranke für die Halomasse aus der Bewegung des entferntesten Kugelhaufens geschlossen wurden. Das beste Ergebnis erzielten sie gar für eine Halomasse von 0.

Die Galaxie NGC 1052-DF2 im Vergleich zur lokalen Gruppe. Auf der X-Achse der Halbmesser in Parsec (1 pc = 3,26 LJ), auf der y-Achse die Geschwindigkeitsdispersion der Kugelhaufen und die Größe der Kreise gibt die Masse an (Zahlen rechts sind Zehnerpotenzen der Sonnenmassen, also 10 ≘ 1010 = 10 Milliarden Sonnenmassen.

Die Galaxie NGC 1052-DF2 im Vergleich zur lokalen Gruppe. Auf der X-Achse der Galaxien-Halbmesser in Parsec (1 pc = 3,26 LJ), auf der y-Achse die Geschwindigkeitsdispersion der Kugelhaufen und die Größe der Kreise gibt die Masse an (Zahlen rechts sind Zehnerpotenzen der Sonnenmassen, also 10 ≘ 1010 = 10 Milliarden Sonnenmassen). NGC 1052-DF2 (schwarz) liegt abgeschlagen in einer einsamen Ecke. Bild: [1]

NGC 1052-DF2 ist mithin keine gewöhnliche UDG. Sie passt nicht ins Schema einer sternarmen Galaxie, die frühzeitig ihr Gas eingebüßt hat. Sie ist einfach nur dünn mit Sternen besetzt. Eine normale Galaxie entsteht, nach allem was wir wissen, aus einer kollabierenden Wolke Dunkler Materie, die sichtbare Materie mit sich zieht. Wie soll eine Galaxie entstehen, die (fast) keine Dunkle Materie besitzt? Autor van Dokkum äußerte sich in einem Interview ratlos:

Keine Theorie hat diese Art von Galaxien vorhergesagt. Die Galaxie ist ein totales Mysterium, an ihr ist alles ungewöhnlich. Wie man so ein Ding erzeugt, ist vollkommen unbekannt.

Ethan Siegel hat einen eigenen Artikel darüber geschrieben und gleich 5 Möglichkeiten aufgezählt, wie man so ein Ding dann doch erzeugen könnte. Hier nur ganz kurz aufgezählt:

  1. Das Gas der Galaxie kann mit Gas im intergalaktischen Raum kollidiert und abgebremst worden sein, während die DM davon nichts gemerkt hat und weiter geflogen ist; solche Fälle kennt man z.B. vom Bullet Cluster. Aus dem ausgebremsten Gas kann dann die ausgedünnte Galaxie entstanden sein.
  2. Bei Kollisionen von Galaxien kann Gas weggeschleudert werden, aus dem eine baryonische Galaxie entstehen könnte.
  3. Die von Quasaren – das sind Galaxien in deren Zentrum einer supermassereiches Schwarzes Loch gerade supermassiv gefüttert wird – ausgestoßenen Jets können zu kleinen Galaxien werden. Da diese von den Magnetfeldern um das Schwarze Loch beschleunigt werden, sollten sie keine Dunkle Materie mehr enthalten, die Elektromagnetismus nicht spürt.
  4. Es wurde vor ein paar Jahren entdeckt, dass manche Galaxien von Wolken aus ionisiertem Gas begleitet werden (Prototyp war Hanny’s Voorwerp), aus denen später eigene Galaxien werden könnten.
  5. Dunkle Materie existiert nicht und verschiedene Galaxien haben einfach verschiedene Zusammensetzungen (hmm, eher nicht).

 

Und was bedeutet das für MOND?

Die beliebteste alternative Erklärung von Gegnern der Dunklen Materie ist eine Modifizierung der entfernungsabhängigen Schwerkraft, genannt “MOdifizierte Newtonsche Dynamik”, kurz MOND. Damit möchte man die Rotationskurven von Galaxien erklären. Auch die Bewegung von Galaxien in Galaxienhaufen. Aber die Theorie berücksichtigt in fast allen Varianten nicht die Relativitätstheorie (sie verletzt sogar die starke Äquivalenz von träger und schwerer Masse) und die einzige Variante, die relativistisch ausgelegt war, ging neulich mit dem Nachweis, dass Gravitationswellen sich genau so schnell fortpflanzen wie elektromagnetische Wellen, den Bach runter. Über die Entstehung primordialen Gases oder das Leistungsspektrum der Hintergrundstrahlung kann MOND im Gegensatz zur Dunklen Materie gar nichts aussagen. Die Dunkle Materie ist einfach viel erklärungsmächtiger, deshalb wird sie von der Mehrheit der Astrophysiker und Kosmologen nicht so leicht aufgegeben.

Was bedeutet NGC 1052-DF2 nun für MOND? Nichts Gutes. Was man daran merkt, dass manche MOND-Anhänger wütend werden und manche versuchen, das Problem wegzulachen bzw. ins Lächerliche zu ziehen. Wenn von “Religion” der anderen Seite die Rede ist, muss der Frustrationslevel schon sehr hoch sein. Die Argumentation von van Dokkum ist natürlich nicht “ich sehe nichts, also existiert es”.

Sondern: leuchtende Materie und dunkle gravitative Masse sind zweierlei separate Dinge, die in beliebigem Mischungsverhältnis auftreten können.

Mal als “Dark Matter Galaxy”, mal als gewöhnliche mit großem DM-Halo, und jetzt mal als fast reine leuchtende Materie, die ganz wunderbar nach unMOdifizierter Newtonscher Dynamik funktioniert. Egal, wie sie entstanden ist. Und deshalb muss es beide geben. Denn wenn die Schwerkraft, wie auch immer ihre entfernungs- und masseabhängige Formel lauten möge, nur an der leuchtenden Masse und ihrer Verteilung hängen würde, dann könnten die UDGs nicht so verschieden sein wie NGS 1052-DF2 und Dragonfly 44.

Und zu den Kritikpunkten aus dem Osterhasen-Artikel:

  • Dass die Kugelhaufen alle in einer Ebene kreisen, haben van Dokkum et al. untersucht und widerlegt, s.o.
  • Das Problem mit den zu hellen Kugelhaufen haben die Autoren ebenfalls erwähnt und ein separates Paper daraus gemacht, deswegen haben sie zwei unabhängige alternative Entfernungsmessmethoden verwendet, die übereinstimmende Ergebnisse liefern. Wenn die Galaxie tatsächlich um mehr als die Hälfte näher wäre und die Kugelhaufen normal hell, wäre sie schwerer, aber nicht um einen signifikanten Faktor, der den Mangel an der 400-fachen Halo-Materie wett machen könnte. Dafür wäre sie dann gemäß ihrer Rotverschiebung mit einer Eigengeschwindigkeit von 1200 km/s unterwegs, was ziemlich ungewöhnlich wäre.
  • Wenn der “externe Feldeffekt” durch die nahe elliptische Galaxie NGC 1052 eine Rolle spielen soll, dann müsste das vorgerechnet werden, um zu überzeugen.
  • Und wenn’s wirklich ein Messfehler wäre: van Dokkum hat versprochen, weitere solche Galaxien zu suchen. Wenn’s bei nur einer bliebe, wäre die Kritik möglicherweise gerechtfertigt, aber wenn noch weitere gefunden werden, dann hat MOND ein echtes Problem. Wir werden sehen.

 

Referenzen

[1] Pieter van Dokkum et al. “A Galaxy Lacking Dark Matter.” Nature. March 29, 2018; arXiv:1803.10237.

Kommentare (117)

  1. #1 Karl Mistelberger
    5. April 2018

    It doesn’t matter how beautiful your theory is, it doesn’t matter how smart you are. If it doesn’t agree with experiment, it’s wrong.

    Das gilt nicht nur für schöne Theorien sondern auch auch für schiache wie MOND.

  2. #2 Oliver Gabath
    5. April 2018

    So schief kam mir MOND an sich nie vor. Ist halt ein alternativer Ansatz und die Zeit wird’s zeigen. Man kann ja auch nicht gerade behaupten, dass die wichtigsten Proponenten ausgewiesene Cranks seien. Aber ich habe bisher nicht verstanden, wie der Effekt physikalisch motiviert wird, wenn nicht durch Wirkung von Masse.

    Eine Form von Materie, die nicht elektromagnetisch wechselwirkt, aber gravitativ? Warum nicht. Erklärt ja auch eine Menge.

    Wodurch wird in MOND die Modifikation des Gravitationsgesetzes motiviert? Reicht Anpassung des bisherigen Wissens oder brauchte man eine fünfte Kraft?

  3. #3 Karl Mistelberger
    5. April 2018

    However, while MOND does a good job explaining the observations, it has the unappealing property of requiring an “interpolation function”. This function is necessary to get a smooth transition from the regime in which gravity is modified (at low acceleration) to the normal gravity regime, which must be reproduced at high acceleration to fit observations in the solar system. In the literature one can find various choices for this interpolation function.

    https://backreaction.blogspot.co.at/2018/03/modified-gravity-and-radial.html

  4. #4 Alderamin
    5. April 2018

    Man kann ja auch nicht gerade behaupten, dass die wichtigsten Proponenten ausgewiesene Cranks seien.

    Sicher nicht. Die Idee an sich ist ja nahe liegend und auch nicht schräger, als Dunkle Materie oder gar Dunkle Energie. Es ist nur eben eine Frage, welche Theorie die besseren Vorhersagen macht und die Beobachtungen am besten beschreibt (vielleicht sind ja auch beide falsch? Gibt ja auch noch Verlinde). Da sucht jetzt jede Seite die Datenpunkte, um die andere zu falsifizieren. So geht Wissenschaft und das ist gut so!

    Aber ich habe bisher nicht verstanden, wie der Effekt physikalisch motiviert wird, wenn nicht durch Wirkung von Masse.

    Genau durch Masse, eben wie bei Newton, nur mit anderer Entfernungsformel.

    Eine Form von Materie, die nicht elektromagnetisch wechselwirkt, aber gravitativ? Warum nicht. Erklärt ja auch eine Menge.

    Wie die Neutrinos. Aber die sind zu schnell (heiß) und zu wenige/zu leicht. Jedenfalls die gewöhnlichen Elektron-, Myon- und Tauon-Neutrinos. Heiße Teilchen können sich nicht zu Galaxien-Gerüsten verdichten, sondern fliegen nur wild durch die Gegend.

    Wodurch wird in MOND die Modifikation des Gravitationsgesetzes motiviert? Reicht Anpassung des bisherigen Wissens oder brauchte man eine fünfte Kraft?

    Durch nichts physikalisches, das ist einfach ein Fit auf die Rotationskurven der Galaxien. Von einer fünften Kraft ist hier nicht die Rede. Die sogenannte Quintessenz ist(war) einer der Kandidaten für die Dunkle Energie, aber mittlerweile favorisiert man da ja die von der ART abgedeckte kosmologische Konstante.

  5. #5 Alderamin
    5. April 2018

    Verlinde, sag’ ich ja. Wobei der mir (bis jetzt) noch zu hoch ist. Vielleicht schreibt mal einer einen verständlichen Artikel darüber.

    Galaxienrotationskurven erklären ist einfach. Aber die Peaks in der Hintergrundstrahlung, die großräumige Struktur des kosmischen Netzes und die Häufigkeiten der Elemente im primordialen Gas müssen auch erklärt werden. So viel ich weiß kann das bisher nur die Dunkle Materie.

  6. #6 stone1
    5. April 2018

    Oh ja, Verlinde: Schwerkraft als emergentes Phänomen im Gegensatz zu einer fundamentalen Grundkraft. In die Richtung haben wir bei Joseph Kuhn spekuliert, weil man über “Gott” nicht wirklich sinnvoll reden konnte. Ein Artikel darüber wäre sehr willkommen.

    Dass es für MOND schlecht aussieht wundert mich nicht, diese Theorie war mir von Anfang an nicht wirklich geheuer. Ist wohl nur eine Frage der Zeit, bis man weitere derart DM-arme oder gar DM-lose Galaxien findet.

    Aber was war nun eigentlich mit dem “Osterhasen-Artikel” gemeint? Eh die Arbeit von van Dokkum oder hab ich was verpasst?

  7. #7 Alderamin
    5. April 2018

    Ein Artikel darüber wäre sehr willkommen.

    Ja, aber nicht von mir… 😉

    Aber was war nun eigentlich mit dem “Osterhasen-Artikel” gemeint?

    Der Link unter “das Problem wegzulachen”: https://prosaderphysik.wordpress.com/2018/03/30/osterhase-nicht-gefunden-existenz-des-osterhasen-bewiesen/

    [Update: habe ihn unter dem “Osterhasen-Artikel” nochmal verlinkt.]

    Ist, soweit ich verstanden habe, von Oliver Müller.

  8. #8 Mars
    5. April 2018

    ein sehr schöner beitrag, du kommst immer gut auf eine noch überschaubare länge und schneidest dennoch so viel an – toll!
    vorallem, weil du auch auf die alternativen thesen eingehst, auch wenn man spürt …. nicht immer ganz objektiv – aber das ist hier, denke ich, ok.

    in dem Osterhasen-link hab ich folgendes gefunden:
    “” Die Leuchtkraft zu messen ist einfach, aber wie messen wir genau die Masse? Dafür beobachten wir die Bewegung der Objekte innerhalb der Galaxie, z. B. der Sterne oder des Gases. Wir sehen, wie sich ein Objekt bewegt, und mittels Netwonschem Gravitationsgesetz leiten wir die Masse her.””

    und genau da gibt es doch ein problem, wenn wie bei MOND angenommen wird, dass die Gravitation irgendwann nur noch 1/r abnimmt
    wenn man das annimmt, dann würden doch die ganzen massen (die man ja aufgrund Newton misst) nicht stimmen, und so das konzept wieder durcheinander bringen
    aber wie könn(t)en wir denn sonst die Gravitation in einer sehr weiten entfernung prüfen und mit der vergleichen, die direkt in der nähe messbar sind ???

    oh, da dreht sich wieder alles um und in meinem kopf
    das schwarze – nicht direkt nachweissbare – zeugs macht mich immer ganz dusselig …..
    trotzdem gerne mehr.

  9. #9 Alderamin
    5. April 2018

    weil du auch auf die alternativen thesen eingehst, auch wenn man spürt …. nicht immer ganz objektiv

    So gut ich sie verstanden habe, aber natürlich kenne ich mich mit DM weitaus besser aus als mit MOND, über DM habe ich ja viel mehr gelesen. Ich bin aber nicht derjenige, der am Ende entscheidet, welche Theorie die bessere ist, ich kann nur die Punkte für und wider die jeweilige Theorie aufzählen. Die lange Ausschlussliste für die DM habe ich ja auch oben angeführt.

    und genau da gibt es doch ein problem, wenn wie bei MOND angenommen wird, dass die Gravitation irgendwann nur noch 1/r abnimmt
    wenn man das annimmt, dann würden doch die ganzen massen (die man ja aufgrund Newton misst) nicht stimmen, und so das konzept wieder durcheinander bringen
    aber wie könn(t)en wir denn sonst die Gravitation in einer sehr weiten entfernung prüfen und mit der vergleichen, die direkt in der nähe messbar sind ???

    Gar nicht nach MOND, denn da stecken ja Konstanten drin, die aus der beobachteten Bewegung abgeleitet werden, das ist ein Daten-Fit. MOND trifft einfach die Annahme, dass die leuchtende Masse gleich der gravitativen ist. Die leuchtende ist relativ leicht zu messen (über die Masse-Leuchtkraft-Funktion). Ich weiß auch gar nicht, ob unter MOND noch gilt, dass man ein kugelförmiges Objekt auf seine gesamte Masse im Schwerpunkt vereint reduzieren darf, wenn man sich außerhalb seines Radius befindet. Ich denke, das gilt unter MOND nicht mehr, und eine große Hohlkugel wäre innen auch nicht mehr kräftefrei.

    Wie kann man MOND überprüfen? Ich habe dazu ein 10 Jahre altes Papier gefunden, anscheinend von MOND-Proponenten. Sollte man wirklich mal ausprobieren. Wäre was für Falcon Heavy 1 gewesen.

    Ansonsten kann man eben nur Fälle vergleichen, bei denen die leuchtende Materie offenbar von gleicher Größenordnung ist, aber die Gravitation verschieden, und genau das tut van Dokkum in der hier betrachteten Arbeit. Wenn er keinem Messfehler unterliegt, hat er, denke ich, MOND widerlegt. Schon Dragonfly 44 hat das im anderen Extrem getan, und da gibt’s keinen mutmaßlichen externen Feldeffekt durch eine nahe Nachbargalaxie.

    Aber auch wenn MOND widerlegt sein sollte, heißt das nicht, dass DM zwingendermaßen richtig ist.

    trotzdem gerne mehr.

    Es kommen noch zwei Sternbild-Artikel, um das Wintersechseck abzuhaken, und dann habe ich schon ein, zwei neue Forschungsartikel im Kopf. Und danach kommt was gaaanz anderes. Nicht einmal Astronomie. So geht ungefähr der Plan, aber es kann ja immer was aktuelles dazwischen kommen.

  10. #10 Mars
    5. April 2018

    … jetzt hast du mich aber fast noch mehr ins trudeln gebracht ..

    bei MOND geht man doch davon aus, dass sich mit einem bestimmten abstand die gravitation nicht mehr – wie im nahen bereich – mit 1/r² sondern eben nur noch mit 1/r abnimmt.

    dabei würden doch – wenn man die schlüsse aus Mond nutzt – eben doch andere massen berechnet werden.
    denn ich hab (noch) keine andere rechnungsgrundlagen gefunden, die eben weit entfernte systeme massemässig erfassen – ausser durch deren bewegung.
    wenn aber die Gravitation ab einem gewissen mass anders in die rechnung einzusetzen wäre …. ja dann …

    ja, bleibt ein spannendes thema in 25 jahren wissen wir hoffentlich mehr.
    aber ich freu mich auch auf den rest der wintersternbilder.
    danke

  11. #11 Alderamin
    5. April 2018

    bei MOND geht man doch davon aus, dass sich mit einem bestimmten abstand die gravitation nicht mehr – wie im nahen bereich – mit 1/r² sondern eben nur noch mit 1/r abnimmt.

    So ähnlich, aber doch anders. Da stecken zwei Stellknöpfe drin, a0 und die µ-Funktion. Die geben einem die Freiheit, die Formel an die Beobachtungen anzupassen.

    dabei würden doch – wenn man die schlüsse aus Mond nutzt – eben doch andere massen berechnet werden.

    Sicher kann man mit der Formel (gegeben bestimmte a0 und µ(x)) eine Masse aus der Bewegung eines beobachteten Objekts bestimmen, aber wenn da was gänzlich anderes als die leuchtende Masse rauskäme, hätte MOND sich ja selbst falsifiziert. Bzw. würde man dann als MOND-Anhänger vermutlich die Masse-Leuchtkraft-Funktion anzweifeln, denn die hängt von der Zusammensetzung der Sternenpopulation ab. Noch mehr Stellknöpfe.

  12. #12 stone1
    5. April 2018

    @Alderamin

    Aha, ja der Blog Prosa der Physik scheint von O. Müller zu sein, aber ganz klar wird das dort nicht.

    Ja, aber nicht von mir…

    Wieso denn jetzt wieder nicht von dir, da stand doch in Kommentar #5 …oops verlesen, da steht tatsächlich schreibt mal einer

    Ich bin ohnehin auf der Suche nach einem Thema für den nächsten Schreibwettbewerb, bis jetzt hab ich nur das Exciton, dass sich zwar vom Namen her “aufregend” anhört, aber doch eher nur in eine sehr kleine Nische passt. ; )
    Hmm ich will nichts versprechen, aber nachdem ich mich für das Thema selbst sehr interessiere, such ich vielleicht ein paar Infos zu Verlindes Theorie zusammen. Eventuell kann ich das sogar bis zum Sommer (wo vermutlich der nächste Schreibbewerb bei Florian stattfindet) in Artikelform gießen, sofern ich das überhaupt ansatzweise verstehen kann. Mal sehen.

  13. #13 rolak
    5. April 2018

    ganz klar wird das dort nicht

    Ach herrjeh, ich hab schon wieder derart reale Hallus .-.-. nee, da isses doch. Das ‘Über‘, unter dem ‘Auf mseiner Uni Webseite’ gelandet werden kann.

  14. #14 Captain E.
    5. April 2018

    Tja, Oliver Müller ist halt sehr von dieser Hypothese überzeugt. Ich glaube, ich nenne ihn ab sofort “Dr. MOND”. 😉

    Der Vorwurf, die Anhänger der Dunklen Materie würden Annahmen treffen, wie es ihnen in den Kram passt, ist aber leider auch der Hauptvorwurf an MOND. “Wir gehen davon aus, dass die Gravitation genau so funktioniert, dass sie mit der leuchtenden Materie genau zu den beobachteten Bewegungen führt.” Ja, ich weiß, es ist ein wenig komplizierter, aber letzten Endes läuft es genau darauf hinaus.

  15. #15 Alderamin
    5. April 2018

    Tja, Oliver Müller ist halt sehr von dieser Hypothese überzeugt.

    Wäre er das nicht, dann wäre er auch der Falsche für das Thema (und es ist gut, dass sich auch damit Forscher auseinander setzen).

    “Wir gehen davon aus, dass die Gravitation genau so funktioniert, dass sie mit der leuchtenden Materie genau zu den beobachteten Bewegungen führt.”

    Es gab da mal jemand, der hat gesagt, tun wir doch einfach mal so als wenn die Lichtgeschwindigkeit für jeden bewegten Beobachter gleich groß erscheint. Und hat Erfolg damit gehabt. Von einer Annahme ausgehen und sie dann zu verifizieren oder falsifizieren zu versuchen ist so verkehrt ja nicht. Man muss nur mitbekommen, ob das Pferd noch lebt, das man gerade reitet. 😉

  16. #16 Alderamin
    5. April 2018

    Uiii, Boxen und Karate kann er. Da muss ich ja vorsichtig mit Kritik sein… 😀

    Nee, Quatsch, in den Kommentaren zu seinem Artikel im Schreibwettbewerb fand ich ihn ganz umgänglich, und großen Respekt für seine Veröffentlichung in Science. Bin gespannt, wann er hier vorbei schaut und mich an die Wand argumentiert.

  17. #17 stone1
    5. April 2018

    @rolak
    Ahja, diesen Link zu seiner Baseler Uniwebsite hatte ich übersehen. Scheint mir heute öfters zu passieren, muss die Fönwetterlage sein die meine Konzentration beeinträchtigt.

  18. #18 Karl Mistelberger
    5. April 2018

    > #16 Alderamin, 5. April 2018
    > Man muss nur mitbekommen, ob das Pferd noch lebt, das man gerade reitet.

    Muss man eben nicht unbedingt. Nach der Meinung einiger versucht sich unter anderen die Politik in der Kunst, auf toten Pferden zu reiten.

  19. #19 Oliver Müller
    Canberra
    6. April 2018

    Dr. MOND hier. 😛 Oder wie wäre es mit Captain MOND?
    Sorry, ich bin gerade im Stress, da ich meine Dissertation in einem Monat abgebe und derzeit in Australien bin, deshalb wollte ich eigentlich nicht auf diesen Artikel eingehen (aber da ich hier jetzt ein paar Mal erwähnt worden bin, fühle ich mich doch ein wenig verpflichtet).

    Zu deinen Kritikpunkten an meinem Artikel (sehr kurz, sorry):
    Punkt 1: Die Rotation der Kugelsternhaufen kann man nur beobachten, falls sie entlang der Sichtlinie ist. Ist diese aber z.B. senkrecht zur Sichtlinie, werden wir diese nicht messen können, also ist Punkt 1 von dir nicht korrekt, da van Dokkum und co. dies gar nicht erst messen können, wie sollen sie es da widerlegt haben?

    Punkt 2: Die Surface Brightness Fluctuation Methode ist extrem ungenau, wenn man nicht die Stellarpopulation der Galaxie kennt. z.B. gibt es ein Paper von meinem Doktorvater (der diese Methode im übrigen mitentwickelt hat), in der die Abweichung zur genaueren TRGB Methode 50% war (Jerjen & Rejkuba 2001, https://arxiv.org/abs/astro-ph/0105144). Die andere Messmethode im Artikel ist über den Hubblefluss (also die Geschwindigkeit der Galaxie), was auch sehr ungenau ist. z.B. wenn du mein Paper hier anschaust (https://arxiv.org/abs/1802.08657, letzte beiden Tabellen), siehst du, dass Geschwindigkeit und Distanz teilweise überhaupt nicht übereinstimmen. Das kommt durch Eigengeschwindigkeiten zustande.

    Punkt 3: Ich habe ja geschrieben, dass wir die Rechnung in ein Fachjournal (also in Nature) eingereicht haben. Mal schauen ob sie den Fehler im Paper also korrigieren. Ich denke aber nicht, dass Nature daran Interesse hat. Wir waren mit van Dokkum in Kontakt, und er hat zugegeben, falsche Annahmen getroffen zu haben, als er die MOND Vorhersage berechnete. Das ist natürlich sofern blöd, da es jetzt in Nature steht, und alle dies als MOND Falsifizierung herannehmen. Aber so oder so, ein Fachartikel zur genauen Berechnung wird noch veröffentlicht, so lange müsst ihr mir glauben, dass es konform ist. Ich denke es ist aber wissenschaftlicher, wenn es zuerst in einem Fachartikel erscheint, und erst nachträglich auf einem Blog.

    Punkt 4: Ja genau, “Extraordinary claims need extraordinary evidence”, was hier nicht der Fall ist. Das Problem ist, dass die vielen UDGs bisher alle Dunkle Materie reich waren, warum also genau dieser eine keine Dunkle Materie hat, ist wirklich fraglich. Vorallem wenn er keine DM hat, wie kann er stabil sein? So wurde ja DM ursprünglich (Ostriker und co.) herangezogen: Falls Galaxien nur Newtonsch wären (also ohne dieses DM Halo), wären sie nicht stabil und würden ausseinandergerissen.

    Du zitierst mich ja auch und sagst, ich verstehe es falsch, dass van Dokkum nicht gesagt hat, “Dunkle Materie existiert.” Das ist aber genau ein Zitat von ihm: “Here, “the absence of dark matter is evidence of its existence,” van Dokkum says. “There is no way around it. It can be in a galaxy or not in a galaxy, but it is not a field, or some alternative thing that manifests itself rather than being a substance.”” (https://www.scientificamerican.com/article/astronomers-boggle-at-a-distant-galaxy-devoid-of-dark-matter/). Ich denke meine Osterhasenanalogie passt also doch ganz gut. 🙂

    Du schreibst auch, dass es nur eine einzige Möglichkeit der relativistischen MOND gibt und diese den Bach runterging. Es gibt mehrere relativistische MOND Theorien, und nicht jede wurde durch GW widerlegt.
    Auch eine Variante, MOND und DM in Einklang zu bringen ist Superfluid Dark Matter (https://backreaction.blogspot.com.au/2018/01/superfluid-dark-matter-gets-seriously.html). Hier ist aber der falsche Ort für so eine Diskussion.

    Da hier in den Kommentarspalten Verwirrung über den Externe Feld Effekt und MOND allgemein herrscht: MOND kommt nur zur Anwendung, wenn man bei kleinen Beschleunigungen ist, das nennt man das “deep-MOND Regime”. Setzt man nun eine solche Galaxie neben eine massive Galaxie wie NGC1052, so hebt das Gravitationspotential der grossen Galaxie die Beschleunigung ins “nicht-MOND Regime” an, also ins Newtonsche, was ja gerade gemessen wurde! Sprich, eigentlich ist NGC1052-DF2 gerade eine Bestätigung für MOND. Wer hätte das gedacht? Wenn Interesse besteht, kann ich mal etwas für Nicht-Experten zum EFE schreiben, dies aber sicher erst nach meiner Diss.

    Warum gerade die Kritik kommt, das MOND einen fit an Galaxien darstellt, ist mir schleierhaft. Hat man die MOND Konstante einmal gemessen (also bei einer Galaxie), ist sie für immer fix. Eine Abweichung der MOND Konstante in einer anderen Galaxie würde MOND widerlegen. Das ist also kein Fit. Hingegen wird ein fit bei jeder Galaxie ja genau mit DM gemacht. Man misst die Masse die man sieht, die Masse die erwartet wird, und füllt die Diskrepanz mit DM auf.

    Ups, eigentlich wollte ich nicht so viel schreiben, und wie gesagt, bin ich wohl mehr ab als anwesend.

    Vielleicht hat mein Blogwettbewerb Artikel ein falsches Bild von mir erzeugt: zu 99% arbeite ich auf Dunkler Materie und nicht MOND (ich selbst habe noch nie ein MOND Paper veröffentlicht). Ich sehe mich einfach sehr offen gegenüber alternativen Theorien und meiner Meinung nach sollten vernünftige Theorien parallel studiert werden.

  20. #20 Captain E.
    6. April 2018

    @Oliver Müller:

    Tja, der (noch nicht ganz) Dr. MOND höchstpersönlich! Dir fällt aber schon auf, dass du zumindest in deinen Blogbeiträgen die sattsam bekannte Linie von Einstein-Leugnern und ähnlichen Dunning-Krugers fährst, ja? Deine eigenen Argumente sind eindeutig und unwiderlegbar/unwiderlegt, aber die der Gegenseite sind immer nur Scheinargumente. Das macht deine Position nicht wirklich glaubwürdig. Aber veröffentlich ruhig deine(n) Fachartikel – dann sehen wir ja weiter. Vielleicht kannst du den einen oder anderen Kollegen überzeugen. So oder so, dumme Fachartikel sind trotz Peer-Review schon zuhauf veröffentlicht worden, und das wird auch weiterhin geschehen – und das gilt für jede mögliche Hypothese.

    Aber vielleicht geschieht es am Ende doch so, dass MOND nur eine andere Schreibweise dafür ist, wie Dunkle Materie den Raum krümmt. In der frühen Quantentheorie hat es so etwas doch schon gegeben, oder? Man hat sich heftigst gestritten, welche von zwei Hypothesen wohl die richtige sei, und am Ende kam heraus: Beide!

    Übrigens, um auf meinen “Lieblingsaspekt” bei diesem Thema zurück zu kommen, an alle und vor allem den Hausherren die Frage: Sehe ich das richtig, dass man zurzeit davon ausgeht, dass Neutrinos ihre Energie und Geschwindigkeit nicht verlieren können, ohne dabei zerstört zu werden? Tun sie es aber nicht, können sie ja auch keinen signifikanten Anteil an der Dunklen Materie darstellen. Was wäre aber, wenn es einen bislang völlig unbekannten Effekt gäbe, der Neutrinos die Energie zerstörungsfrei entzöge? Es könnte dann viel mehr geben als bislang angenommen, und es könnte dann für die Dunkle Materie reichen. Das Problem: Wie wollte man kalte Neutrinos nachweisen, falls es sie gäbe? Der Nachweis gelingt ja nur dadurch, dass sie schwach mit anderer Materie wechselwirken und dabei die in ihnen enthaltene Energie einbringen.

  21. #21 Oliver Müller
    6. April 2018

    @Captain E
    Hast du dir meinen Blog auch wirklich angeschaut?

    Hier schreibe ich über Ram-Pressure Stripping durch Dunkle Materie https://prosaderphysik.wordpress.com/2017/07/15/ram-pressure-stripping-belaestigung-am-galaktischen-arbeitsplatz/,

    hier wie man Dunkle Materie vermisst https://prosaderphysik.wordpress.com/2017/10/01/die-vermessung-der-welt-aka-der-dunkler-materie/,

    hier wie ein Sternhaufen entdeckt wurde
    https://prosaderphysik.wordpress.com/2018/02/24/die-entdeckung-von-gaia-i-oder-wenn-der-baum-den-wald-verdeckt/

    hier über Zwerggalaxien und Dunkle Materie
    https://prosaderphysik.wordpress.com/2014/02/09/zwerggalaxien-gleich-und-gleich-gesellt-sich-gern/

    Das ist alles fachlich fundiert und nach Standardmodell.

    Im Artikel “Ist das kosmologische Prinzip widerlegt?”
    https://prosaderphysik.wordpress.com/2017/01/04/ist-das-kosmologische-prinzip-widerlegt/
    (auf der ja die ART basiert) relativiere ich ja genau die (/meine) Argumente und sage, dass die Analyse wohl auf einem Fehler im Katalog zurückzuführen ist. Also so eindeutig/unwiderlegbar schreibe ich ja wirklich nicht, wie du mich darstellen willst.

    Ich weiss echt nicht, warum du mich hier die ganze Zeit angreifen und diffamieren willst. Bitte lass das doch in Zukunft sein.

    Was sagen die anderen Kommentatoren hier dazu, ist das ein angemessener Umgang?

  22. #22 Alderamin
    6. April 2018

    So oder so, dumme Fachartikel sind trotz Peer-Review schon zuhauf veröffentlicht worden, und das wird auch weiterhin geschehen – und das gilt für jede mögliche Hypothese.

    Bitte freundlich bleiben. 🙂 Bin gerade ein wenig im Stress, ich antworte gleich auf Oliver und auch auf Deine Mail.

  23. #23 Captain E.
    6. April 2018

    @Alderamin:

    Bitte freundlich bleiben. 🙂 Bin gerade ein wenig im Stress, ich antworte gleich auf Oliver und auch auf Deine Mail.

    Ich sagte ja, dass das für jede mögliche Hypothese gilt. Wenn Pieter van Dokkum jetzt tatsächlich bereits zugegeben hat, einen Fehler gemacht zu haben, der den Prüfern auch nicht aufgefallen ist, dann wäre das genau so ein Fall.

  24. #24 Alderamin
    6. April 2018

    Sorry, ich bin gerade im Stress, da ich meine Dissertation in einem Monat abgebe und derzeit in Australien bin, deshalb wollte ich eigentlich nicht auf diesen Artikel eingehen (aber da ich hier jetzt ein paar Mal erwähnt worden bin, fühle ich mich doch ein wenig verpflichtet).

    Schön, dass Du die Zeit gefunden hast, mal reinzuschauen!

    Zu deinen Kritikpunkten an meinem Artikel (sehr kurz, sorry):
    Punkt 1: Die Rotation der Kugelsternhaufen kann man nur beobachten, falls sie entlang der Sichtlinie ist. Ist diese aber z.B. senkrecht zur Sichtlinie, werden wir diese nicht messen können, also ist Punkt 1 von dir nicht korrekt, da van Dokkum und co. dies gar nicht erst messen können, wie sollen sie es da widerlegt haben?

    Ich schreib’s mal so, dass es alle verstehen: man kann von der Raumgeschwindigkeit der Kugelhaufen nur diejenige messen, die entlang der Sichtlinie verläuft (Radialgeschwindigkeit, auf den Beobachter zu oder von ihm weg). Habe ich oben auch erwähnt. Nur im Falle, dass die Bewegung vollkommen senkrecht zur Blickrichtung erfolgt, sieht man gar keine Radialgeschwindigkeit, also etwa, wenn man exakt senkrecht von oben auf eine gemeinsame Bahnebene aller Kugelhaufen schauen würde, was statistisch ziemlich unwahrscheinlich ist. Besteht ein Winkel der Bahnebene (Inklination i) ungleich 90° zur Sichtlinie, dann sieht man die Geschwindigkeit um den Faktor sin(i) verkleinert. Das ist soweit klar, auch van Dokkum.

    Die Analyse der Geschwindigkeitsdispersion geht von zufällig verteilten Bahnen und Inklinationen aus. Nicht zufällig wäre aber, da hast Du Recht, eine gemeinsame Bahnebene aller Kugelhaufen, die Du (wenn ich das recht verstanden habe) bei der Galaxie Centaurus A gemessen hast (gibt’s das Science-Paper auf arXiv? Oder magst Du hier mal einen Gastartikel darüber schreiben?). Auch für die Milchstraße behauptest Du dies in Deinem “Osterhasen-Artikel”, gibt’s da auch eine Arbeit zu? Nach allem, was ich gelernt habe, sind die Umlaufbahnen der Kugelhaufen der Milchstraße zufällig und unsortiert, sie umschwirren das Milchstraßenzentrum wie ein Bienenschwarm auf unabhängigen elliptischen Umlaufbahnen. Obwohl eine bevorzugte Bahnebene nicht völlig verwunderlich wäre, die Milchstraße ist ja eine scheibenförmige Galaxie. Wie gesagt, kenne ich nur anders, kann aber auf altem Wissensstand sein.

    Jedenfalls hat van Dokkum eine Bewegung messen können, die Inklination ist also < 90°, wir schauen schlimmstenfalls auf eine leicht gegen die Senkrechte verkippte mutmaßliche gemeinsame Bahnebene. Was er oben im vorletzten Bild geplottet hat, sind die Geschwindigkeiten der Kugelhaufen mit ihren Orten (oder sagt man hier auch Örtern?) rechts und links bzw. oben und unten der Galaxie. Wenn eine gemeinsame Bahn vorläge, würden alle Kugelhaufen in der gleichen Ebene und im gleichen Drehsinn kreisen, dann wäre die Geschwindigkeit auf einer Seite der Galaxie eher positiv (von uns weg) und auf der anderen eher negativ (auf uns zu). Das gibt der Plot aber nicht her. Folglich kann man eine gemeinsame Ebene hier ausschließen.

    Punkt 2: Die Surface Brightness Fluctuation Methode ist extrem ungenau, wenn man nicht die Stellarpopulation der Galaxie kennt.

    Das ist klar, aber das Alter der Galaxie kann man doch aus der Farbe abschätzen und daraus, dass sie kein nennenswertes Gas enthält, aus dem nach der Entstehung noch Sterne hätten hervorgehen können (wie elliptische Galaxien). Der Farbindex war, glaube ich, im Paper vermerkt.

    Das Ergebnis hat auch in der Tat eine größere Fehlerabweichung als das mit der Rotverschiebung gemessene, aber die beiden Ergebnisse liegen in gemeinsamen Fehlerbalken, das stützt die Entfernungsmessung. Und wie gesagt, die Kugelhaufenhelligkeit liefert mit nur 8 Mpc ein Ergebnis, das eine unsinnig hohe Eigenbewegung der Galaxie impliziert, weil ein Großteil der Rotverschiebung dann nicht kosmologisch wäre. Daher sind die sich wechselseitig stützenden Entfernungsbestimmungen mit der SRB-Methode und dem kosmologischen Doppler plausibler. Und wie gesagt, selbst wenn die Galaxie 3mal näher wäre, machte dies nicht den um einen Faktor 400 zu kleinen Wert der Halo-Materie wett. Dann wäre man immer noch 2 Größenordnungen drunter.

    Punkt 3: Ich habe ja geschrieben, dass wir die Rechnung in ein Fachjournal (also in Nature) eingereicht haben. Mal schauen ob sie den Fehler im Paper also korrigieren.

    Kannst sie ja auf jeden Fall auf arXiv legen.

    Ich denke aber nicht, dass Nature daran Interesse hat. Wir waren mit van Dokkum in Kontakt, und er hat zugegeben, falsche Annahmen getroffen zu haben, als er die MOND Vorhersage berechnete.

    Dann bitte ihn doch, das Statement öffentlich zu wiederholen oder dass Du den Austausch (Mail?) veröffentlichen darfst. Was genau hat er denn zugegeben falsch vorausgesetzt zu haben?

    Punkt 4: Ja genau, “Extraordinary claims need extraordinary evidence”, was hier nicht der Fall ist. Das Problem ist, dass die vielen UDGs bisher alle Dunkle Materie reich waren, warum also genau dieser eine keine Dunkle Materie hat, ist wirklich fraglich. Vorallem wenn er keine DM hat, wie kann er stabil sein? So wurde ja DM ursprünglich (Ostriker und co.) herangezogen: Falls Galaxien nur Newtonsch wären (also ohne dieses DM Halo), wären sie nicht stabil und würden ausseinandergerissen.

    Dazu müsste man mal die Geschwindigkeitsdispersion der Sterne selbst messen, da müsste der Virialsatz doch perfekt greifen, sind ja viele Sterne. Dass 10 Kugelhaufen keine tolle statistische Basis bieten, ist ja unzweifelhaft, aber das sind halt die, die man hat. Frage mich, warum er das (Sterne vermessen) nicht schon gemacht hat.

    Du zitierst mich ja auch und sagst, ich verstehe es falsch, dass van Dokkum nicht gesagt hat, “Dunkle Materie existiert.” Das ist aber genau ein Zitat von ihm: “Here, “the absence of dark matter is evidence of its existence,” van Dokkum says. “There is no way around it. It can be in a galaxy or not in a galaxy, but it is not a field, or some alternative thing that manifests itself rather than being a substance.”” (https://www.scientificamerican.com/article/astronomers-boggle-at-a-distant-galaxy-devoid-of-dark-matter/). Ich denke meine Osterhasenanalogie passt also doch ganz gut. 🙂

    Nein, weil das Statement von van Dokkum nur Sinn macht im Zusammenhang damit, dass die DM, also der Osterhase, in anderen Galaxien sehr wohl seine Anwesenheit verrät. In Sky & Telescope stand, wenn man sonst nur Tassen mit Kaffee kannte, hat man jetzt den Kaffee alleine gefunden, was beweist, dass die Tasse ein eigenständiges Objekt ist.

    Du schreibst auch, dass es nur eine einzige Möglichkeit der relativistischen MOND gibt und diese den Bach runterging. Es gibt mehrere relativistische MOND Theorien, und nicht jede wurde durch GW widerlegt.

    Hattte mal irgendwo gelesen, dass nur TeVes relativistisch sei.

    Auch eine Variante, MOND und DM in Einklang zu bringen ist Superfluid Dark Matter (https://backreaction.blogspot.com.au/2018/01/superfluid-dark-matter-gets-seriously.html). Hier ist aber der falsche Ort für so eine Diskussion.

    Sagt die denn voraus, dass Licht und Gravitationswellen gleich schnell sein sollen? Ist dafür nicht gerade die Verletzung des starken Äqivalenzprinzips das Problem?

    Da hier in den Kommentarspalten Verwirrung über den Externe Feld Effekt und MOND allgemein herrscht: MOND kommt nur zur Anwendung, wenn man bei kleinen Beschleunigungen ist, das nennt man das “deep-MOND Regime”.

    Ja, zwischen Newton und MOND gibt’s dann über die µ-Funktion eine Interpolation. Das Blöde ist, µ ist wieder so ein Fit, den man passend machen kann. Es wäre erst zu zeigen, dass die anderweitig kalibrierte Funktion hier exakt das gewünschte Ergebnis liefert. Problem ist aber auch, dass Dragonfly 44 extrem viel offenbar dunkle Masse hat, wie bekommt man die beiden unter den gleichen Hut? Sie sehen von außen sehr ähnlich aus, DF44 ist etwas größer.

    Setzt man nun eine solche Galaxie neben eine massive Galaxie wie NGC1052, so hebt das Gravitationspotential der grossen Galaxie die Beschleunigung ins “nicht-MOND Regime” an

    Wieso das, -DF2 ist doch im freien Fall. Sogar das von mir in #9 verlinkte Paper nimmt an, dass man die abweichend MOND-Gravitationsbschleunigung messen kann, wenn man eine Sonde mit dem Experiment im freien Fall oberhalb des Sonnensystems aussetzt, oder in einem der Lagrangepunkte, wo sich die Schwerkraft von Erde und Sonne aufheben. Nach Deiner Argumentation störte dann ja die Gravitation von Sonne und Milchstraße.

    Wenn Interesse besteht, kann ich mal etwas für Nicht-Experten zum EFE schreiben, dies aber sicher erst nach meiner Diss.

    Sehr gerne! Würde mich über einen Gastartikel freuen.

    Warum gerade die Kritik kommt, das MOND einen fit an Galaxien darstellt, ist mir schleierhaft. Hat man die MOND Konstante einmal gemessen (also bei einer Galaxie), ist sie für immer fix. Eine Abweichung der MOND Konstante in einer anderen Galaxie würde MOND widerlegen. Das ist also kein Fit.

    Mit der Interpolationsfunktion hast Du schon zwei Stellschrauben, was einen Fit für mehrere Galaxienarten erlaubt (so wie 3 Linsen in einem Apochromaten 3 Farben an den gleichen Ort fokussieren können, während 2 Linsen im Achromaten das nur für 2 Farben können, mal als ein praktisches Beispiel eines Fits) und da elliptische Galaxien und Kugelhaufen mehr oder weniger kugelförmig sind und Spiralgalaxien einen kugelförmigen DM-Halo haben sollen, sollten die durch DM bestimmten Geschwindigkeiten relativ ähnlich sein, da passt dann aber auch der gleiche Fit einigermaßen. Die Nagelprobe wäre hingegen eine Anwendung auf eine Galaxie wie Dragonfly 44 und im Vergleich auf NGC 1052-DF2, siehe oben.

    Hingegen wird ein fit bei jeder Galaxie ja genau mit DM gemacht. Man misst die Masse die man sieht, die Masse die erwartet wird, und füllt die Diskrepanz mit DM auf.

    Man ordnet gemessener Bewegung (Sterne, Kugelhaufen) die Masse zu, die Newton da sehen möchte. Oder Einstein, wenn’s um Gravitationslinsen geht. Das ist kein Fit, sondern ein Rückschluss gemäß vollkommen unabhängig gemessener und verifizierter Gesetze, Newton und ART. Und dann fehlt eben die sichtbare Materie zu der Masse, die Differenz nennt man “dunkel” und weiß nicht, was es ist. Ein Fit ist hingegen, das Bewegungsgesetz aus der gleichen Beobachtung abzuleiten, die man damit beschreiben will.

    Vielleicht hat mein Blogwettbewerb Artikel ein falsches Bild von mir erzeugt: zu 99% arbeite ich auf Dunkler Materie und nicht MOND (ich selbst habe noch nie ein MOND Paper veröffentlicht). Ich sehe mich einfach sehr offen gegenüber alternativen Theorien und meiner Meinung nach sollten vernünftige Theorien parallel studiert werden.

    Ist ja gut, wenn man noch ein zweites Pferd im Stall hat, falls das erste sich das Bein bricht und erschossen werden muss, oder falls es verhungert. Aber jedenfalls kommst Du nicht ganz “impartial” rüber (ich aber anscheinend auch nicht, siehe #8). Ich halte mich halt an die Mehrheitsmeinung. Der “Mainstream” liegt meistens richtig. Millionen Fliegen können sich nicht irren 😉 Als zuschauender Amateur kann ich mich aber entspannt zurücklehnen und mit Popcorn genießen, wie Ihr Profis das untereinander ausfechtet.

  25. #25 Alderamin
    6. April 2018

    Was sagen die anderen Kommentatoren hier dazu, ist das ein angemessener Umgang?

    Mag ich auch nicht, der Ton auf Scienceblogs ist mir mitunter auch zu rau. Aber Captain E. sagte ja schon, dass es nicht persönlich gegen Dich gerichtet war. Wer in Science das Titelbild erobert hat, ist über jeden Vorwurf des Cranktums kpc-weit erhaben.

  26. #26 Alderamin
    6. April 2018

    Sehe ich das richtig, dass man zurzeit davon ausgeht, dass Neutrinos ihre Energie und Geschwindigkeit nicht verlieren können, ohne dabei zerstört zu werden? Tun sie es aber nicht, können sie ja auch keinen signifikanten Anteil an der Dunklen Materie darstellen.

    Die Theorie besagt anscheinend nicht, dass sie nicht abgebremst werden können, sie besagt nur, dass man sie in abgebremstem Zustand nicht mehr nachweisen kann, weil sie nur mit fast c mit Materie hin und wieder wechselwirken.

    Das Problem ist vielmehr, dass ihre Ruhmasse unter (und keiner weiß wieviel) 3eV liegt, und damit kommt man in der Theorie nicht auf genug Neutrinos als DM-Kandidaten. Jedenfalls was die bekannten Elektron-/Myon-/Tau-Flavors betrifft (“sterile Neutrinos” sind hingegen ein Kandidat).

  27. #27 Captain E.
    6. April 2018

    @Alderamin:

    Die Theorie besagt anscheinend nicht, dass sie nicht abgebremst werden können, sie besagt nur, dass man sie in abgebremstem Zustand nicht mehr nachweisen kann, weil sie nur mit fast c mit Materie hin und wieder wechselwirken.

    Interessant, aber einen möglichen Wirkmechanismus hat man dafür auch noch nicht, oder?

    Meine Vermutung, dass hypothetische kalte Neutrinos noch weniger wechselwirken als die normalen heißen und daher wesentlich schlechter nachgewiesen werden können, war also schon mal nicht so schlecht.

    Das Problem ist vielmehr, dass ihre Ruhmasse unter (und keiner weiß wieviel) 3eV liegt, und damit kommt man in der Theorie nicht auf genug Neutrinos als DM-Kandidaten. Jedenfalls was die bekannten Elektron-/Myon-/Tau-Flavors betrifft (“sterile Neutrinos” sind hingegen ein Kandidat).

    Von denen man leider absolut nicht sagen kann, ob es sie überhaupt gibt. Tja, die Neutrinos sind halt im Grunde ein guter Kandidat für Dunkle Materie, nur sind sie zu leicht, zu wenige und zu energiereich. Abbremsen ist prinzipiell möglich, nur wie soll das vonstatten gehen?

    Noch eine Überlegung: Die Effizienz einer möglichen Neutrinokühlung (= -bremsung) bestimmt logischerweise, wieviele von ihnen “überleben”. Langsame halten länger durch als schnelle, auf unserem Zeitrahmen betrachtet, weil sie noch weniger mit normaler Materie wechselwirken. Was schneller abkühlt, koppelt sich früher von der baryonischen Materie und der Gefahr der zerstörenden Wechselwirkung ab. Gibt es eigentlich eine Abschätzung, wieviele davon in den berühmten ersten drei Minuten des Universums entstanden sein müssten?

    Aber damit erst einmal Schluss – ist ja sowieso nur wildes Spekulieren! 🙂

  28. #28 Captain E.
    6. April 2018

    @Alderamin:

    Mag ich auch nicht, der Ton auf Scienceblogs ist mir mitunter auch zu rau. Aber Captain E. sagte ja schon, dass es nicht persönlich gegen Dich gerichtet war. Wer in Science das Titelbild erobert hat, ist über jeden Vorwurf des Cranktums kpc-weit erhaben.

    Wohl wahr, und der braucht dann auch keine Verschwörungstheorien zu spinnen, man (der Mainstream”) wolle ihn mundtot machen. Aber natürlich stimmen nicht immer alle Fakten in so einem Artikel, und selbst wenn, müssen die gezogenen Schlussfolgerungen nicht allgemeine Anerkennung finden. Davor schützt auch das an und für sich bewährte Peer-Review-Verfahren nicht.

    Ein Fachartikel in einem angesehenen Magazin kann ja auch niemals das Ende sein, sondern höchstens das Ende des Anfangs. Man hat etwas geschaffen, das für wert befunden wurde, veröffentlicht zu werden, und stellt es hiermit aller (Fach-) Welt zur Verfügung. Ob es falsch, schlecht oder gut ist, wird danach entschieden. Millionen Fliegen…

  29. #29 Alderamin
    6. April 2018

    Interessant, aber einen möglichen Wirkmechanismus hat man dafür auch noch nicht, oder?

    Was meinst Du, wie sie bei c mit Materie wechselwirken? Na, genau umgekehrt, wie sie erzeugt werden (Beta-Zerfall), inverser Beta-Zerfall.

    Von denen man leider absolut nicht sagen kann, ob es sie überhaupt gibt.

    Wie bei allen anderen DM-Kandidaten auch.

    Abbremsen ist prinzipiell möglich, nur wie soll das vonstatten gehen?

    Nee, da habe ich mich schlecht ausgedrückt, abbremsen ist schon schwierig machbar (wenn überhaupt), aber Neutrinos dürften nach der Antwort in dem Link (wusste ich vorher auch nicht) langsam sein können. Die müssten dann halt schon langsam entstanden sein, kurz nach dem Urknall, steht in der Antwort auf physics.stackexchange.com.

    Gibt es eigentlich eine Abschätzung, wieviele davon in den berühmten ersten drei Minuten des Universums entstanden sein müssten?

    Wovon, langsame Neutrinos? Im Link auf physics.stackexchange.com steht etwas von der 50-fachen Menge der solaren Neutrinos (das sind wieder viel, viel mehr als der Durchschnitt im Universum insgesamt, aber frag’ mich nicht, wie hoch der ist). Und hier steht was über den Neutrino-Hintergrund und seine Temperatur. So oder so, selbst die 50-fache Menge der Sonnenneutrinos reicht sicher nicht für die DM, Neutrinos sind ein Hauch von Nichts.

  30. #30 Captain E.
    6. April 2018

    @Alderamin:

    Was meinst Du, wie sie bei c mit Materie wechselwirken? Na, genau umgekehrt, wie sie erzeugt werden (Beta-Zerfall), inverser Beta-Zerfall.

    Oder beinahe c? Sie scheinen ja eine Ruhemasse zu haben, wenngleich sie auch sehr, sehr, …, sehr klein ist.

    Ich bin dir aber gerade nicht ganz sicher. Passiert das wirklich? Bei beiden Betazerfällen werden Elektronen und Neutrinos abgestrahlt bzw. ihre Antiteilchen (oder die Neutrinos sind ihre eigenen Antiteilchen). Kann dieser Vorgang durch die Wechselwirkung eines Neutrinos mit einem Neutron oder Proton angeregt werden? Nun ja, vielleicht schon. Durch die Wechselwirkung beispielweise eines Neutrons mit einem Neutrino könnte eines der beiden Down-Quarks in ein Up-Quark, ein Elektron und ein (virtuelles?) Antineutrino umgewandelt werden, das mit dem Neutrino zu einem Photon zerstrahlt. Das dürfte aber nicht sein, was man seinerzeit beim Betazerfall beobachtet hat. Die EM-Strahlung hätte man beobachten können, aber so war es nicht. Stattdessen kam man darauf, dass ein nicht detektierbares Teilchen entstehen und die fehlende Energiedifferenz davontragen müsse.

    Was ich mir aber eher vorstelle, ist die Entstehung von Teilchen wie Myonen, Pionen, Kaonen usw., die dann umgehend weiter zerfallen. Obwohl, kommt dabei am Ende irgendetwas stabiles heraus außer EM-Strahlung?

    […]

    Nee, da habe ich mich schlecht ausgedrückt, abbremsen ist schon schwierig machbar (wenn überhaupt), aber Neutrinos dürften nach der Antwort in dem Link (wusste ich vorher auch nicht) langsam sein können. Die müssten dann halt schon langsam entstanden sein, kurz nach dem Urknall, steht in der Antwort auf physics.stackexchange.com.

    Ich verstehe! Neutrinos entstehen mit einer bestimmten Geschwindigkeit und behalten diese bis zu ihrer Zerstörung. Bekannt ist nur, dass diese ziemlich dicht bei c liegt. Langsamere sind niemals entdeckt worden, aber es könnte sie halt geben.

    Wovon, langsame Neutrinos?

    Eben jene.

    Im Link auf physics.stackexchange.com steht etwas von der 50-fachen Menge der solaren Neutrinos (das sind wieder viel, viel mehr als der Durchschnitt im Universum insgesamt, aber frag’ mich nicht, wie hoch der ist). Und hier steht was über den Neutrino-Hintergrund und seine Temperatur. So oder so, selbst die 50-fache Menge der Sonnenneutrinos reicht sicher nicht für die DM, Neutrinos sind ein Hauch von Nichts.

    Definitiv! Da müsste es schon zehn hoch ziemlich viele geben, damit es hinhaut. Aber gut, da man ja nicht weiß, was die Dunkle Materie ist und wieviel es davon gibt, ist das nicht so einfach. Wenn man nicht gerade MOND bevorzugt, muss zu einem ziemlich frühen Zeitpunkt im Universum eine Riesenmenge an merkwürdigen Sachen entstanden sein. (Es gibt da die Überlegung, dass die baryonische Materie in die von der Dunklen Materie zuvor gebildeten Schwerkrafttöpfe hineingeflossen ist und somit die ersten Galaxien gebildet hat.) Welcher Prozess hat da was erzeugt? Vielleicht das 50fache der Sonnenneutrinos, aber was noch darüberhinaus?

    Das einfachste wäre ja, wenn der Prozess tatsächlich erheblich mehr an langsamen Neutrinos erzeugt hätte. Die Dinger kennt man zumindest, jedenfalls mit einer wahnwitzigen Geschwindigkeit von beinahe c. Na, überlassen wir auch das den Profis…

  31. #31 Alderamin
    7. April 2018

    @Captain E.

    Bei beiden Betazerfällen werden Elektronen und Neutrinos abgestrahlt bzw. ihre Antiteilchen (oder die Neutrinos sind ihre eigenen Antiteilchen). Kann dieser Vorgang durch die Wechselwirkung eines Neutrinos mit einem Neutron oder Proton angeregt werden?

    Bin zwar kein Experte für Teilchenreaktion, aber doch ja, so funktioniert doch der Nachweis von Neutrinos in den großen Tanks. Beim inversen Betazerfall wird ein Neutrino von einem Proton eingefangen und unter Aussendung eines Positrons wird es zu einem Neutron. Und die Reaktion benötigt ein Neutrino von mindestens 1,8 MeV/c² kinetischer Energie.

    Was ich mir aber eher vorstelle, ist die Entstehung von Teilchen wie Myonen, Pionen, Kaonen usw., die dann umgehend weiter zerfallen. Obwohl, kommt dabei am Ende irgendetwas stabiles heraus außer EM-Strahlung?

    Irgendwas bleibt immer übrig, Erhaltung der Baryonenzahl, Erhaltung der Ladung (gut, Positronen finden immer ein Elektron zur Paarvernichtung, wären ansonsten aber stabil). Gibt viele Zerfallsketten von Teilchen, das eine oder andere Neutrino wird dabei sicher auch frei, aber das ist nicht mein Metier…

  32. #32 Karl Mistelberger
    7. April 2018

    > #20 Alderamin, 6. April 2018
    > Ja, zwischen Newton und MOND gibt’s dann über die µ-Funktion eine Interpolation. Das Blöde ist, µ ist wieder so ein Fit, den man passend machen kann.

    Hat sich nicht Bine über die “Covariant Emergent Gravity” Gedanken gemacht, wobei ohne weitere Annahmen was dafür heraus kam?

  33. #33 Mars
    8. April 2018

    nicht dass ich weiter unruhe in das thema bringen will – dazu ist es viel zu spannend!

    aber in siencXX : https://www.scinexx.de/wissen-aktuell-22600-2018-04-06.html

    wird über einen galaxiehaufen berichtet, bei dem man annahm – ähnlich dem diskussionswürdigen Bullet-Cluster – dass die dunkle materie abgebremst /materie weiterfliegt (oder andersrum je nach wechselwirkung).

    wenn so ein streitpunkt bisher nun von weiterer seite wieder neues licht erhällt, wird dann nicht die eine odere andere these gestärkt richtung ART oder MOND?
    oder werden beide davon (wieder) profitieren – wird es da überhaupt eine ‘lösung’ geben können?

    … und könnte das einer hier evt nochmal aufnehmen
    denn weiterhin kreist mir dunkle materie im kopf, und sollte nochmal sortiert werden.

  34. #34 Alderamin
    8. April 2018

    Du meinst sicher diesen Artikel. Da schreibt sie, dass ihre Theorie keine Interpolationsfunktion braucht. Was die aus den UDGs macht, wäre dann natürlich die Frage (oder eigentlich nicht, denn zwei gleich aussehende, aber anscheinend vollkommen verschiedene Masse besitzende Galaxien wird man ohne DM wohl in keine gemeinsame Rotationskurvengleichung bekommen – falls Oliver nicht über den EFE was gezaubert bekommt).

  35. #35 Alderamin
    8. April 2018

    Sehe ich nicht, dass DM oder MOND davon profitieren, es ist ja erstens nicht so, dass in jedem Galaxienhaufen Materie und DM verschiedene Wege gehen würden, sondern im Regelfall ist das nicht so. Und zweitens gibt es noch mehr Beispiele für kollidierende Galaxienhaufen neben dem Bullet Cluster und Abbell 3827, wo es eben doch der Fall ist. Es fällt halt ein Datenpunkt weg.

    Die im Scinexx-Artikel angesprochene Arbeit ist vielmehr bedeutsam, weil eben jener Galaxienhaufen vor kurzem noch verdächtigt wurde, Annihilierungsstrahlung von DM auszusenden, also die Strahlung die entstehen würde, wenn die DM-Teilchen ihre eigenen Antiteilchen wären und sich gelegentlich bei Kollisionen zu Strahlung paarvernichten würden. Das scheint nun doch nicht der Fall zu sein (was im Scinexx-Artikel darüber steht, dass das Standard-ΛCDM-Modell annehmen würde, dass DM nicht mit sich selbst wechselwirke, würde ich so nicht unterschreiben wollen; vielmehr macht ΛCDM keinerlei Aussage über die Natur der Teilchen überhaupt, außer ihrer Schwerkraft, und berücksichtigt infolgedessen keine Paarvernichtung oder andere Wechselwirkungen, was konkrete Annahmen über die Teilchen voraussetzen würde).

    Die aus dem Zentrum der Milchstraße empfangene Gamma-Strahlung, die als Kandidat für DM-Paarvernichtung galt, könnte nun auch eher auf Pulsare zurückgeführt werden. Es ist halt die Crux, dass man immer nur negative Aussagen über die DM machen kann. Für die WIMP-Theorie sieht es nicht besonders gut aus. Axionen und sterile Neutrinos würden, so viel ich weiß, allerdings keine solche Strahlung verursachen, die Kandidatenkiste ist also noch nicht leer. Der Positivbeweis wurde aber auch noch von keiner Theorie geführt. Es hat halt auch noch niemand ein Experiment wie das oben von mir verlinkte durchgeführt, mit dem man zeigen könnte, dass es eine Abweichung von der Newton-ART-Gravitation gibt. Genauso wenig wie man die mutmaßlichen DM-Partikel gefunden hat. Solange man nur gegenseitig Falsifikationen aufeinander abschießt, wird man das Rätsel nicht endgültig lösen können. Wie gesagt, wenn eine Theorie widerlegt ist, heißt das nicht, dass eine bestimmte andere deswegen notwendigerweise richtig sein muss.

  36. #36 Mars
    9. April 2018

    dank dir für das quantum licht das du in dieses ‘düstere’ thema bringen konntest.

    … dann bleibt es weiter spannend
    und du hast immer wieder ein thema, das du aufgreifen kannst
    supiii

  37. #37 Oliver Müller
    9. April 2018

    Hier ein Statement von van Dokkum persönlich, dass er die MOND Berechnung falsch durchgeführt hat: https://www.pietervandokkum.com/ngc1052-df2

  38. #38 Alderamin
    10. April 2018

    Danke für den Link. Dauerte ein wenig, bis ich Zeit hatte, das Statement komplett und in Ruhe zu lesen.

    Na gut, zuerst sagt er, die Dispersion der Geschwindigkeiten habe aufgrund des Outliers GC98 eine zu kleine Streuung, eigentlich sollte man 20 km/s für das 90%-Konfidenzintervall annehmen (was die Masse der Galaxie dann um maximal einen Faktor 2 anheben würde, wenn ich das richtig verstanden habe). Seine Folgerung:

    Now, all this detailed discussion should not detract from the key result of the paper: however you calculate the confidence limits, the dispersion of the globular clusters is way smaller than that of other Local Group galaxies of the same stellar mass, and of the similar-looking ultra-diffuse galaxy Dragonfly 44. There’s just no question about that

    Mit anderen Worten, auch ein Faktor 2 überbrückt nicht den gewaltigen Massenunterschied zwischen DF2 und DF44, die sich in der Leuchtkraftmasse ähneln.

    Dann schiebt er nach, dass GC98 ohnehin das schlechteste Spektrum habe, was den bestimmten Geschwindigkeitswert in Frage stelle und erklären kann, warum er ein Ausreißer ist.

    Danach schreibt er, er habe immer gedacht, dass MOND kein Problem mit isolierten Galaxien mit großem Anteil an scheinbarer Materie hätte und DF2 ein Problem sei. Wenn man nun den EFE durch die nahe Galaxie NGC 1052 mitbetrachte (die, wenn ich mich recht erinnere, nur 70.000 LJ [oder pc?] von DF2 entfernt ist), sollte nach MOND DF2 in den Newtonschen Bereich rutschen und sich Newtonsch verhalten, was dann zu seinen Messungen passen würde.

    In diesem Fall gebe es aber ein Problem mit DF44:

    In any case, getting back to the question whether MOND is able to fit other galaxies than DF2, the closer MOND gets to explaining NGC1052-DF2, the harder it is to explain Dragonfly 44. This galaxy, as discussed in a 2016 paper, is even larger than DF2, has a similar stellar mass, and a velocity dispersion of 47 +- 7 km/s. It lives in the Coma cluster – I imagine the EFE is important there, also, but even if it isn’t, the dispersion is significantly higher than even the isolated MOND prediction. The key issue for alternatives to dark matter is the difference between NGC1052-DF2 and Dragonfly 44, […]. If a theory is able to fit one object, it will have a hard time fitting the other. This is what we should have said more clearly in the paper – my misunderstanding was that I thought MOND could fit Dragonfly 44.

    Er nimmt seine Kritik an MOND also nicht zurück, sondern betont, dass es eben nicht hinhaue, mit MOND zwei (nach DM) so verschieden massive Galaxien wie DF2 und DF44 mit ähnlicher Leuchtkraft unter einen Hut zu bringen (was ich oben auch schon erwähnte).

  39. #39 Captain E.
    10. April 2018

    Ja, aber er schreibt auch, dass wenn man mittels MOND die Galaxis NGC1052-DF2 gut berechnet bekommt, wird es bei Dragonfly 44 zugleich um so schwerer.

    Und das ist meines Erachtens genau das Problem von MOND, dass man eben versucht, mittels beobachtbarer Materie die Regeln der Gravitation neu zu fassen anstelle es dabei zu belassen, dass diese Regeln bereits sehr gut verstanden sind und man nicht immer alles sehen kann (bzw. sogar den überwiegenden Teil).

    In der Mathematik kann man eine Funktion definieren mit jeder Menge Spezialfällen, aber das geht nicht so einfach in der Physik. Wenn MOND NGC1052-DF2 und Dragonfly 44 nicht gleich gut berechnen kann, taugt sie doch mit Sicherheit nichts, oder? Zumindest tut sie das dann nicht in der aktuell angewendeten Form. Es fehlen dann Parameter, und die könnten in der Dunklen Materie begründet liegen.

  40. […] i die unbekannte Neigung der Bahn-Drehachse gegen die Sichtlinie ist (wie wir das schon bei den Kugelsternhaufen von NGC 1052 DF-2 kennengelernt haben). Die Minimummasse von Thestias bei angenommenem i nahe 90° (Kantenblick auf die Bahn) beträgt […]

  41. #41 Oliver Müller
    11. April 2018

    @Alderamin
    Ich frage mich, warum du in meinem Blogartikel kritisierst, dass ich nichts gerechnet habe (dein 3. Punkt), wenn van Dokkum aber schreibt, DF44 ist ein Problem für MOND – was so nie gerechnet und publiziert wurde – du dies sofort glaubst und als Argument aufgreifst. Sehe ich hier den Bias, den ich in meinem berüchtigten Artikel beschrieben habe? 🙂
    Meine Erwartung war aber, dass hier genau DF44 aufgegriffen wird, zu köstlich. XD

    Hier eine Analyse, die die Methodik von van Dokkum kritisiert: https://medium.com/@samvaughan01/a-galaxy-without-dark-matter-ae18003b87c

    Mein Punkt bleibt der Gleiche: das Paper ist nicht sauber. Morgen (oder spätestens übermorgen) werden drei Artikel von denen ich weiss ins arxiv gestellt, die das Nature Paper kritisieren, von der Analyse bis zur Interpretation. Zwei Artikel kommen dabei aus dem Dunkle Materie Lager.

    Ich glaube, dass ist ein fundamentales Problem des Wissenschaftsjournalismus. Anstelle, dass Dinge als Hypothesen dargestellt werden, sind sie sofort schwarz oder weiss, richtig oder falsch. Vielleicht erinnert sich noch jemand an die Sensationsmeldung “überlichtschnelle Neutrinos”, dass auf ein Kabelkurzschluss zurückzuführen war. Was dies aber für ein Echo in den Medien (und der Wissenschaftsgemeinde) gab… Ich nehme ja schwer an, dass über solche Gegendarstellungen hier und in anderen populärwissenschaftlichen Foren nicht geschrieben wird.

  42. #42 Alderamin
    11. April 2018

    Ich frage mich, warum du in meinem Blogartikel kritisierst, dass ich nichts gerechnet habe (dein 3. Punkt), wenn van Dokkum aber schreibt, DF44 ist ein Problem für MOND – was so nie gerechnet und publiziert wurde – du dies sofort glaubst und als Argument aufgreifst.

    Weil ich den EFE nicht kannte und nicht verstehe – er klingt für mich wie eine aus dem Hut gezauberte Lösung, die sich experimentell ja offenbar nie bestätigen lassen wird, weil wir etwa der Gravitation der Milchstraße nie entkommen können; und freier Fall “rettet” ja anscheinend nicht vor dem Effekt -, aber sehr wohl verstehe, dass zwei Galaxien, die vergleichbar viel leuchtende Masse haben, aber offenbar sehr unterschiedliche gravitative Masse haben, ein Problem für eine Theorie sein müssen, die die Gravitation alleine an der leuchtenden Materie festmachen will.

    Neu ist mir allerdings seit heute morgen, dass Sabine Hossenfelder in ihrer Theorie die Schwerkraft auf Felder zurückführt, die gar nichts mehr mit der vorhandenen leuchtenden Materie zu tun haben müssen. Warum das noch unter MOND läuft, entzieht sich meinem Verständnis. Dunkle Partikel, Dunkle Felder – wo ist da noch der fundamentale Unterschied? Die Partikel kann man ja vielleicht mal finden, aber die Felder?

    Meine Erwartung war aber, dass hier genau DF44 aufgegriffen wird, zu köstlich. XD

    War ja auch offensichtlich, aber weglachen gilt nicht 😛

    Ich glaube, dass ist ein fundamentales Problem des Wissenschaftsjournalismus. Anstelle, dass Dinge als Hypothesen dargestellt werden, sind sie sofort schwarz oder weiss, richtig oder falsch.

    Bin kein Journalismus, nur ein Blog, und versuche den Lesern die Dinge zu erklären, die ich verstanden habe, in dem Fall das Dokkum-Papier und wie er gearbeitet hat (weil ich genau diese Arbeitsweisen in meinem Blog den Lesern näher bringen möchte). Die einzige Replik dazu, von der ich zur Zeit, als ich den Artikel schrieb, wusste, war Deine, und mir ist die Veralberung darin etwas negativ aufgestoßen – enweder hat man gute Argumente, dann braucht’s keinen Zynismus, oder man hat sie nicht und übertüncht das auf diese Weise. Deswegen und weil er bei Florian verlinkt worden war, habe ich den auch aufgegriffen und fand die Argumente für mich nicht alle überzeugend.

    Wenn es berechtigte Kritik gibt, dann darf und soll die auch zu Wort kommen. Ich selbst bin nicht kompetent genug, darüber viel zu schreiben, weil ich keinen Überblick über die MOND-Varianten habe, aber wenn die entsprechenden Leute, die den Durchblick haben, das tun, bin ich der letzte, der ihnen den Mund verbieten würde. Im Gegenteil, ich habe mich mal dagegen empört, dass Ethan Siegel einem Mitarbeiter von Pavel Kroupa (oder ihm selbst, bin nicht mehr sicher) die Veröffentlichung eines MOND-Papers verbieten wollte.

    Vielleicht erinnert sich noch jemand an die Sensationsmeldung “überlichtschnelle Neutrinos”, dass auf ein Kabelkurzschluss zurückzuführen war.

    Was mir damals sofort klar war (nicht, dass es an einem Kabel lag, aber dass die Messung irgendeinen Fehler haben musste).

    Ich nehme ja schwer an, dass über solche Gegendarstellungen hier und in anderen populärwissenschaftlichen Foren nicht geschrieben wird.

    Wie gesagt, ich kann nichts über Dinge schreiben, die ich nicht verstehe und über die ich keinen Überblick habe. Aber ich spreche gerne nochmal meine Einladung aus, wenn Du was darüber schreiben möchtest, würde ich mich über einen Gastartikel freuen. Und wer sich sonst unter den Mitlesern hier berufen und befähigt fühlt, für den gilt das gleiche. Kontrovers diskutieren werden wir das dann am Ende sicher auch. Ich werde mich darum bemühen, dass der Ton der Diskutanten dabei sachlich bleibt.

  43. #43 Oliver Müller
    11. April 2018

    @Alderamin
    Ich habe eine längere Antwort geschrieben, leider ist mir der PC abgestürtzt und alles futsch. Deshalb nur kurz nochmals:

    Die einzige Replik dazu, von der ich zur Zeit, als ich den Artikel schrieb, wusste, war Deine, und mir ist die Veralberung darin etwas negativ aufgestoßen – enweder hat man gute Argumente, dann braucht’s keinen Zynismus, oder man hat sie nicht und übertüncht das auf diese Weise.

    Schwarz-weiss Denken? 😉 Meine Argumente, die 4 Punkte am Schluss, sind alle meiner Meinung nach gute Argumente und zeigen, weshalb der Artikel problematisch sein könnte (Augenmerk hierbei auf könnte). Da habe ich nichts mit Zynismus überdeckt. Auf einem persönlichen Blog nehme ich mir aber das Recht, auch eine persönliche/emotionale Note hinzuzufügen. In der Fachliteratur muss ich genug trocken schreiben. 😛 Nicht jeder mag das, kann ich gut verstehen.

    Ich habe erst jetzt deinen Kommentar #20 gesehen, also du von einer nochmaligen Einladung für einen Artikel geschrieben hast (ich dachte mir, warum schreibst du *nochmals*). Über das plane-of-satellites Paper habe ich jetzt schon auf meinem Blog geschrieben, sorry: https://prosaderphysik.wordpress.com/2018/04/07/das-plane-of-satellites-problem/
    Ich wollte eigentlich noch mehr schreiben, aber die Zeit drängte und ich wollte ihn raushauen. Aber schreibe mir doch eine E-Mail wegen eines allfälligen Artikels. Eventuell auch über MOND (den EFE z.B.) mit weniger Zynismus versteht sich.

    Bei einigen Punkten deines Kommentar #20 muss ich stillschweigen, da ich sonst unveröffentlichte Arbeiten hier ausplaudern würde. Sorry. Und auch sonst ist das mir jetzt zu exzessiv, dass in einer Kommentarspalte zu diskutieren. Nur hier erlitt ich Herzschmerzen:

    Nach allem, was ich gelernt habe, sind die Umlaufbahnen der Kugelhaufen der Milchstraße zufällig und unsortiert, sie umschwirren das Milchstraßenzentrum wie ein Bienenschwarm auf unabhängigen elliptischen Umlaufbahnen. Obwohl eine bevorzugte Bahnebene nicht völlig verwunderlich wäre, die Milchstraße ist ja eine scheibenförmige Galaxie. Wie gesagt, kenne ich nur anders, kann aber auf altem Wissensstand sein.

    Die äusseren Kugelsternhaufen richten sich in einer Ebene aus, in der sie um die Milchstrasse rotieren Quelle: https://adsabs.harvard.edu/abs/2012MNRAS.423.1109P
    . Wie die Zwerggalaxien! Dies ist das sogenannte planes-of-satellites Problem, auf dem ich ja arbeite.

  44. #44 Alderamin
    11. April 2018

    Ich habe eine längere Antwort geschrieben, leider ist mir der PC abgestürtzt und alles futsch.

    Oh, das ist ärgerlich, tut mir leid. Kenne ich vom iPad. Tippe mir einen Wolf im Kommentarbereich und dann lädt Safari einfach mal ohne Grund die Seite neu, und der getippte Text ist futsch. Selten, dass ich dann nochmal von vorne anfange.

    Meine Argumente, die 4 Punkte am Schluss, sind alle meiner Meinung nach gute Argumente und zeigen, weshalb der Artikel problematisch sein könnte

    Auf die ich ja dann auch eingegangen bin.

    Über das plane-of-satellites Paper habe ich jetzt schon auf meinem Blog geschrieben, sorry:

    Danke für den Link, schaue ich mir mal an.

    Aber schreibe mir doch eine E-Mail wegen eines allfälligen Artikels. Eventuell auch über MOND (den EFE z.B.) mit weniger Zynismus versteht sich.

    Mail ist raus und vielen Dank im Voraus für den Artikel!

    Bei einigen Punkten deines Kommentar #20 muss ich stillschweigen, da ich sonst unveröffentlichte Arbeiten hier ausplaudern würde. Sorry.

    Du machst es spannend :-), aber klar, verstehe ich.

    Die äusseren Kugelsternhaufen richten sich in einer Ebene aus, in der sie um die Milchstrasse rotieren Quelle:

    Vielen Dank für den Link!

  45. […] findet sich, ausgehend von einem Verhältnis von 2 Sonnenmassen auf eine Sonnenleuchtkraft (vgl. Artikel zum Dokkum-Paper) an im normalen Sternenlicht leuchtender Materie 0,54±0,08 und 0,58±0,09 Billionen Sonnenmassen […]

  46. #46 Alderamin
    12. April 2018

    @Oliver

    Ok, habe mir Deine Seite mal durchgelesen, die für sich betrachtet sehr überzeugend klingt, auf der Basis dass die statistischen Angaben bzgl. der Simulationen stimmen. Selbst Ethan Siegel hat sich mit Eurer Arbeit beschäftigt und weist sie nicht völlig zurück, was schon einiges zu bedeuten hat, wenn man ihn kennt.

    Ich habe mir dann das Pawlowski-Kroupa-Paper mal angeschaut und wer das so alles aufgegriffen hat. Da findet sich eine ganze Reihe von Artikeln, die keine Unverträglichkeit zwischen mehr oder weniger häufigen Satellitengalaxie-Scheiben und der DM-Theorie finden. Hier mal eine Auswahl:

         A new spin on disks of satellite galaxies

         The alignment of satellite galaxies and cosmic filaments: Observations and simulations

         A comparison of the distribution of satellite galaxies around Andromeda and the results of ΛCDM simulations

         The spatial distribution of galactic satellites in the Λ cold dark matter cosmology

         The velocity shear tensor: Tracer of halo alignment

    Simulationen sind ja auch nur vereinfachende Modelle der Realität mit beschränkter Auflösung und unter Auslassung mancher Effekte (z.B. der Rückwirkung von Supernovae auf das einfallende Gas, was das Missing-Satellite-Problem lösen könnte).

    Dann sieht die Sache nicht mehr so eindeutig aus.

    Wenn eine Simulation die Realität sehr gut beschreibt, kann man daraus folgern, dass eine ganze Kette vom Modell über die Anfangsbedingungen, Korrektheit der Simulation, Auflösung, Berücksichtigung der relevanten Effekte etc. offenbar korrekt war.

    Wenn aber die Simulation wesentliche Aspekte der Realität nicht richtig beschreibt, heißt das nicht notwendigerweise, dass das simulierte Modell falsch war, sondern vielmehr, dass irgendwo in der o.g. Kette mindestens ein Fehler gewesen sein muss. Mit den o.g. Papers löst sich vielleicht das Disk-of-Satellites-Problem auch in Wohlgefallen auf.

    Wie ich schon mal erwähnte, mich hat immer gewundert, warum die Milchstraße zu einer Spiralgalaxie geworden ist, obwohl sie durch den Einfang zahlreicher kleiner Galaxien gewachsen sein soll. Wenn der Einfang nicht irgendwie gehäuft in der gleichen Ebene erfolgt wäre, hätte sie doch eigentlich als elliptische Galaxie enden müssen. Deswegen wundert mich die Anordnung der Satellitengalaxien in einer Scheibe weniger, als dass die Simulationen dies nicht schon hergeben. Spricht eher gegen die Simulationen.

    Wie Du am Ende Deines Artikels sagst, es bleibt spannend.

  47. #47 Karl Mistelberger
    mistelberger.net
    13. April 2018

    > #43 Oliver Müller, 11. April 2018
    > Ich habe eine längere Antwort geschrieben, leider ist mir der PC abgestürzt und alles futsch.

    Hatte nicht schon Pierre de Format mit ähnlichen Problemen zu kämpfen?

    Apropos: Meinen PC kann ich ziemlich problemlos neu starten, z.B. nach einem Systemupdate. Die Verbindung zum Server sowie das in Bearbeitung befindliche Formular sind immer noch da. Möglicherweise liegt es an ihrem steinzeitlichen Apparat.

  48. #48 Alderamin
    13. April 2018

    Das Science-Papier von Oliver Müller et al. findet sich übrigens auf arXiv. Werde ich mir mal in Ruhe zu Gemüte führen.

    https://arxiv.org/abs/1802.00081

  49. #49 Uli Schoppe
    18. April 2018

    Ich kann mir nicht helfen, ich lese mich immer mehr ein und MOND kommt mir deutlich mehr als Frickeltheorie rüber als DM.
    Wenn Gastartikel dann wenn es geht so das ich mir nicht mal erst wieder ein WE Mathematik reinpumpen muss auf das meine Familie mich für crank hält, das wäre schön

  50. #50 Oliver Müller
    19. April 2018

    Ich kann dieses Buch (leider nur auf Englisch) sehr empfehlen, dass auf gute Art erklärt, wie man auf die Idee der Dunklen Materie kam, was die heutigen Problem sind, und wie dies unter dem Aspekt von MOND erscheint.
    “The Dark Matter Problem: A Historical Perspective” von Prof. Robert Sanders, emeritierter Professor am Kapteyn Institute in Groningen.

    https://www.amazon.de/Dark-Matter-Problem-Historical-Perspective/dp/1107677181

    Ich denke, MOND kann man sehr schön beschreibend erklären, ohne grosse Mathematik. Also, no worries.

  51. #51 Alderamin
    25. April 2018

    @Oliver Müller

    Die äusseren Kugelsternhaufen richten sich in einer Ebene aus, in der sie um die Milchstrasse rotieren Quelle: https://adsabs.harvard.edu/abs/2012MNRAS.423.1109P
    . Wie die Zwerggalaxien! Dies ist das sogenannte planes-of-satellites Problem, auf dem ich ja arbeite.

    Dazu gab es eben (heute ist GAIA-Tag!) ein neues Paper (schick, was für ‘ne Präsentation!).

    Aus dem Abstract:

    Results. Our limited and straightforward analyses have allowed us for example to (i) determine absolute and very precise proper motions for globular clusters; (ii) detect clear rotation signatures in the proper motions of at least five globular clusters; (iii) show that the satellites of the MilkyWay are all on high-inclination orbits, but that they do not share a single plane of motion

    Und eine andere Arbeit aus 2017 behauptet frech:

    There is No Missing Satellites Problem.

    We show that there is a match between the observed satellite counts corrected by the detection efficiency of the Sloan Digital Sky Survey (for luminosities L≳ 340 L⊙) and the number of luminous satellites predicted by CDM, assuming an empirical relation between stellar mass and halo mass. The “missing satellites problem”, cast in terms of number counts, is thus solved

  52. #52 Oliver Müller
    25. April 2018

    Hier eine Visualisierung der Orbits: https://www.astro.rug.nl/~ahelmi/research/dr2-dggc/
    erstellt vom Erstautor des Artikels. Und hier noch ein Zitat von der Webseite:

    The second movie shows that two dwarfs (Ursa Minor and Draco) share very similar orbits and that many of the dwarfs, but not all, lie in a common fairly planar configuration.

    Ob die Orbits in einer oder mehreren Ebenen liegen, ist weniger relevant, als die allgemeine Abflachung der Orbits. Nach LCDM ist eine isotrope Verteilung erwartet, GAIA hat aber eine Anhäufung senkrecht zur Milchstrassenspirale gefunden, entsprechend der vast polar structure. Also sind die Orbits nicht isotrop verteilt, was für das plane-of-satellites Problem spricht.

    Hier aus dem Paper:

    We find that their orbits tend to be perpendicular
    to the Galactic disc (the majority cluster at an inclination
    of ∼ 90 ± 20 deg) but span a broad range of orientations.

    Ich bin mir nicht sicher, ob es Proper Motions für die äusseren Kugelsternhaufen der plane-of-satellites gibt. Im Artikel sieht dies eher nach den nahen Kugelsternhaufen aus, die wie erwartet nicht planar sind. Aber da muss ich mich genauer informieren.

    Und des weiteren, auch aus dem Artikel, was auf ein weiteres Problem für LCDM verweist:

    . Secondly,
    the eccentricity distribution of the dwarfs is inconsistent with the
    predictions of cosmological simulations, where satellites are expected
    to be on rather radial orbits (e.g. Barber et al. 2014).

    Ich würde mal sagen: es bleibt spannend!

  53. […] Milchstraße, sondern auch in den Magellanschen Wolken, in 87 Kugelsternhaufe und in 984 Zwerg- und ultradiffusen Zwerggalaxien gefunden. Die Autoren bestimmten unter anderem für ca. 54000 RR-Lyrae-Sterne die interstellare […]

  54. #54 Nicht von Bedeutung
    Hameln
    26. Mai 2018

    “Man kann also aus der Schwerebeschleunigung auf die Masse eines Himmelskörpers schließen.”

    Diese Annahme ist mMn nicht ganz korrekt, zumindest stimmt sie genau dann nicht mehr, wenn ein Kern auf mehr als ein kugelsymetrisches Objekt pro Orbit Einfluß nimmt, wie es in vielen Galaxien (außer halt bisher in NGC 1052-DF2) der Fall ist. Wenn sich diese Massen gleichmäßig auf dem Orbit verteilen, beschleunigt der Kern nämlich überhaupt nicht mehr und die Beschleunigungen, die durch die Kraft hervorgerufen werden, verteilen sich auf die umlaufenden Objekte. Kurz gesagt, müsste man im Kern ein Schwarzes Loch annehmen (Division durch 0) und auf den Orbits eben Dunkle Materie. Man sieht deutlich, dass die Orbits in Galaxien deutlich höher belegt sind, als in Sonnensystemen, dass sich die Anzahl verklumpter Elementarteilchen ändert, jedoch nicht. Dennoch erwartet man dort Kepler-Kurven und in Folge dessen Dunkle-Materie-Teilchen. Wenn das alles so zutrifft, wie es gesagt wird, wieso erwartet man dann keine Kepler-Kurven in starren rotierenden Scheiben? Hat man sich denn wenigstens mal die Graphen Winkelgeschwindigkeit zu Orbitradius verschiedener umlaufender Systeme angesehen? Hängt die Rotationsgeschwindigkeit eines Orbits um den Kern vllt. von seiner prozentualen Belegung ab? Gilt C=4π²/(v²r) für alle umlaufenden Systeme und C=t²/r³ nur für Systeme mit Kepler-Orbits? Ist sich die Wissenschaft sicher, dass sie Masse 100%ig verstanden hat?

  55. #55 Nicht von Bedeutung
    Hameln
    26. Mai 2018

    Zusatz: Für meine Hypothese müssen sich die Orbits nicht einmal in einer Ebene bewegen. Für die Berechnung ihrer Umlauf- bzw. Winkelgeschwindigkeiten genügt die Information, wie hoch die Strecke des Umfangs mit Massen belegt ist.

  56. #56 Alderamin
    27. Mai 2018

    , zumindest stimmt sie genau dann nicht mehr, wenn ein Kern auf mehr als ein kugelsymetrisches Objekt pro Orbit Einfluß nimmt,

    Solange die Massen der Objekte im Orbit klein gegen die der Zentralmasse sind, spielen sie für die mittlere kinetische Energie der anderen Objekte keine Rolle. Wie bei den Planeten im Sonnensystem.

    Wenn sich diese Massen gleichmäßig auf dem Orbit verteilen, beschleunigt der Kern nämlich überhaupt nicht mehr und die Beschleunigungen, die durch die Kraft hervorgerufen werden, verteilen sich auf die umlaufenden Objekte.

    Diesen Satz verstehe ich nicht. Welcher Kern, welche Beschleunigung, wieso sollen sich die Massen der Kugelhaufen gleichmäßig verteilen und was soll das mit ihrer Bewegung zu tun haben? Nach dem Gravitationsgesetz kreist jede Masse für sich um den Massenschwerpunkt aller Massen, der bei vernachlässigbarer Satellitenmasse mit dem Schwerpunkt der Zentralmasse annähernd zusammenfällt. Das ist bei Kugelhaufen, die eine Galaxie umkreisen, der Fall. Eine besondere Anordnung ist nicht erforderlich.

    Kurz gesagt, müsste man im Kern ein Schwarzes Loch annehmen (Division durch 0) und auf den Orbits eben Dunkle Materie.

    Die Kugelhaufen umkreisen kein Schwarzes Loch. In der Milchstraße hat z.B. das zentrale Schwarze Loch eine Masse von 4 Millionen Sonnenmassen, die Milchstraße insgesamt aber von hunderten Milliarden. Die Kugelhaufen umkreisen den Schwerpunkt der Gesamtmasse der Milchstraße, der zwar mit dem Schwarzen Loch zusammenfällt, aber von der Bahngeschwindigkeit her sieht man, dass das Schwarze Loch keine Rolle für die Anziehungskraft spielt.

    Man sieht deutlich, dass die Orbits in Galaxien deutlich höher belegt sind, als in Sonnensystemen,

    Was heißt das „höher belegt“? Wir reden hier nicht von Elektronenorbits unter Einfluss der Schrödingergleichung. Es gibt unendlich viele Orbits um die Milchstraße und jedes Objekt, das um sie kreist, hat einen eigenen, abhängig von Ort und Geschwindigkeit des Objekts.

    Dennoch erwartet man dort Kepler-Kurven und in Folge dessen Dunkle-Materie-Teilchen. Wenn das alles so zutrifft, wie es gesagt wird, wieso erwartet man dann keine Kepler-Kurven in starren rotierenden Scheiben?

    Abgesehen davon, dass Galaxien keine starren Scheiben sind, hat eine kugelförmige Massenverteilung ein anderes Schwerefeld als eine scheibenförmige, d.h. die Bahngeschwindigkeit verhält sich etwas anders mit zunehmendem Abstand vom Schwerpunkt und ob man sich in der Scheibenebene befindet oder nicht. Das gilt insbesondere für die Sterne in der Scheibe und ihre Umlaufgeschwindigkeit in Abhängigkeit vom Radius der Bahn. Weit außerhalb der Scheibe gleicht sich das Schwerefeld dem einer punktförmigen Masse immer mehr an. Um die Größenordnung der Galaxienmasse festzustellen, ist der Abstand der Kugelsternhaufen schon groß genug. Keplerorbits sind nicht gegeben, wenn ein Objekt sich in die Galaxie hineinbewegt. Bei kugelförmiger Massenverteilung kann man zeigen, dass nur die Masse innerhalb des Radius des Objekts vom Massenschwerpunkt wirksam ist, die außerhalb kompensiert sich weg (siehe Link oben, Gauss’s Theorem, S.20), d.h. auf dem Weg nach innen nimmt die anziehende Masse scheinbar ab. Kepler-Orbits gibt es nur dann, wenn das Objekt in der Scheibenebene auf einer Kreisbahn kreist, oder gänzlich außerhalb der Galaxie und ihres DM-Halos bleibt, denn dann bleibt die im Radius eingeschlossene Masse konstant.

    Hat man sich denn wenigstens mal die Graphen Winkelgeschwindigkeit zu Orbitradius verschiedener umlaufender Systeme angesehen?

    Das sind die berühmten Rotationskurven von Galaxien. Das Gravitationsgesetz ist gut verstanden und was heraus kommen müsste für bestimmte Massenverteilungen steht z.B. oben im Link. Insofern kann man aus der Bewegung auf die Verteilung und Menge der Masse rückschließen. MOND und andere sehen dass zwar etwas anders, müssen dann aber erklären können, wieso Galaxien mit vollkommen verschiedenen Mengen an leuchtenden Sternen offenbar ähnliche Schwerkräfte aufbringen können (und was die DM in der Frühphase des Universums und der Galaxienentstehung ersetzen soll – da braucht man sie nämlich auch, sonst funktionieren die Modelle zur Erzeugung von Helium und Lithium bzw. zur Galaxienentstehung nicht).

    Hängt die Rotationsgeschwindigkeit eines Orbits um den Kern vllt. von seiner prozentualen Belegung ab?

    Nein. Wie soll man überhaupt festlegen, wie stark „belegt“ ein Orbit ist? Wie breit ist ein Orbit, wo fängt der nächste an? Nein, jedes Objekt hat seine individuelle Umlaufbahn um den Massenschwerpunkt aller anderen. Störungen können auftreten, wenn sich zwei Objekte stark nähern, aber zwischen den Sternen ist sehr viel Platz, das passiert selten. Wenn man eine Momentaufnahme einer Galaxie betrachtet, wird man solche Fälle nicht beobachten (höchstens einzelne Sterne, die mit hoher Geschwindigkeit durch die Galaxie sausen, weil sie aus einem Mehrfachsystem herausgeschleudert wurden, oder die Reste verschluckter Zwerggalaxien, die noch als Sternströme mit ähnlicher Geschwindigkeit und Richtung unterwegs sind).

    Ist sich die Wissenschaft sicher, dass sie Masse 100%ig verstanden hat?

    In der Wissenschaft ist man sich nie über irgend etwas völlig sicher, man hat halt überprüfte(!!!) Theorien, die die Welt bestmöglich erklären. Wenn aber eine Theorie durch eine andere ersetzt werden soll, dann muss die neue in allen Aspekten gleichwertig und in mindestens einem besser sein (die Beobachtungen besser erklären; rechnerisch, mathematisch, nicht etwa im Sinne von „hört sich besser an“). Das hat MOND nach Meinung der meisten Astronomen noch nicht geleistet.

  57. #57 Nicht von Bedeutung
    27. Mai 2018

    Hallo Alderamin
    Das Alles, was du schreibst und das Alles, was du nicht verstehst, hat mit der Frage zu tun, ob auch Wissenschaftler Gravitation komplett verstanden haben. Du widerholst auch nur, was wissenschaftlich “feststeht” (wofür es sicher einen Grund gibt) und genau da liegt mMn das Problem. Soweit ich das sehe, lohnt es sich beim Thema Gravitation aber, wenn sich jeder abweichend vom sog. Mainstream seine Gedanken macht – unabhängig davon, ob er es kann oder nicht.
    Die Rotationskurven mit Winkelgeschwindigkeit zum Radius unterscheiden sich gravierend von den Rotationskurven mit Umlaufgeschwindigkeit zum Radius. Die “Kurve” von starren Scheiben ist darin nämlich eine horizontale Linie und alle Anderen (Kepler, flache Rotation und Galaxien) entwickeln sich kurvenförmig nach unten (Grafik). MMn erkennt man an diesen Kurven die Abhängigkeit von Rotationsverhalten und Dichte eines umlaufenden Systems am besten.
    Ich jedenfalls bin mir gar nicht mal so sicher, dass weitere Orbitmassen für die Rotation keine Rolle spielen und schon gar nicht bei der Menge, die in Galaxien auftaucht. Erde und Mond z.B. rotieren um einen gemeinsamen Schwerpunkt, das Baryzentrum. Kopiert man den Mond und platziert ihn um 180° versetzt auf dem selben Orbit des Vorhandenen, verlagert sich das Baryzentrum des gesamten Systems in den Erdmittelpunkt, was zur Folge hat, dass die Erde nun ruht. Auf die Erde wirkt nun zwei mal die Anziehungskraft der Körper in entgegengesetzte Richtung und hinzu kommt die Kraft, die zwischen den beiden Monden wirkt. Diese Kraft ist nun für die Beschleunigungen verantwortlich, mit denen sich die Monde um die Erde bewegen und man kann sicher sein, dass es andere (höhere) sind, als jene, mit welchen die Massen der Monde und der Erde bestimmt wurden. Je mehr Massen nun darauf Einfluss nehmen, desto höher wird die Rotationsgeschwindigkeit und bisher ist noch nicht ein einziges Teilchen Dunkle Materie hinzugekommen – es haben sich nur Kräfte und Beschleunigungen definierter Massen geändert. Dunkle Materie findet sich mMn deswegen genau in jenen Dingen, die Wissenschaftler als irrelevant betrachten.

    Nach dem Gravitationsgesetz kreist jede Masse für sich um den Massenschwerpunkt aller Massen

    ist demnach eine vollkommen falsche Einschätzung und genau so sieht es bei der Messung der Gravitationskonstanten aus. In Cavendish-Experimenten wird nämlich ausschließlich nur der Fall “Eine Erde und ein Mond” – also zwei Massen betrachtet. Für umlaufende Systeme aber gilt allgemein Massen von Partikeln im Medium, z.B. Sterne in Galaxien oder Gipspulver, dass man gemächlich bis zur vollständigen Aushärtung in rotierendes Wasser schüttet.
    Und was MoND angeht – da bin ich auch kein Fan von, denn erstens kann kein Mensch wissen, ob dass, was wir im Universum sehen tatsächlich schon alles ist und zweitens wird auch dort die Gravitationskonstante verwendet und deren Herkunft ist mMn aus den oben genannten Gründen mehr als fragwürdig.
    Theoretisch kann ich meine Thesen sogar berechnen, nur leider habe ich als Laie kaum oder gar keinen Zugang zu wissenschaftlichen Beobachtungen, z.B. finde ich keine Stelle im Netz, wo Galaxien auf ihre Dichte hin untersucht werden und deswegen bringe ich auch keine spruchreifen Formeln hervor – nur theoretische. Z.B.
    t\propto r^x
    bzw.
    C=\frac{t^2}{r^x}
    wobei sich x aus
    x=-E(\frac{\pi}{2}, f(n))\cdot (1-f(n))
    ergibt, wobei E() das vollständige elliptische Integral zweiter Art und f(n) eine Funktion für die prozentuale Belegung des Orbits ist.
    f(n)=n
    passt jedenfalls nicht, weil sich daraus Flache Rotationskurven bereits bei etwa 34% Dichte einstellen – zuviel mMn.

  58. #58 Nicht von Bedeutung
    27. Mai 2018

    Ooops, da hat sich ein Flüchtigkeitsfehler eingeschlichen…
    Es muss
    C=\frac{t^2}{r^{2x}}
    heissen.

  59. #59 Alderamin
    27. Mai 2018

    Das Alles, was du schreibst und das Alles, was du nicht verstehst, hat mit der Frage zu tun, ob auch Wissenschaftler Gravitation komplett verstanden haben.

    Was heißt “komplett verstanden”? Ich erinnere mich an eine Physik-Grundvorlesung in Aachen, wo einer der Entdecker der Bosonen der schwachen Wechselwirkung einen Gastvortrag hielt (weiß nur noch, dass es ein Italiener war, könnte Carlo Rubbia gewesen sein). Er sagte “I’ll show you an experiment they [die Physiker] can’t explain.” Und dann ließ er einen Stift fallen. Was er meinte war, dass die Physik keinen Grund dafür angeben kann, dass Massen sich anziehen. Man kann zwar genau beschreiben, was passiert und das wahlweise mit einem Kraftfeld, mit gekrümmter Raumzeit oder Austauschteilchen mit Spin 2 erklären, aber die zwingende Ursache, warum Masse anziehend sein muss, kann niemand aufführen. Physik ist eine beschreibende Wissenschaft und kann Dinge nur auf andere, grundlegendere Beschreibungen zurück führen. Die Gravitation ist eine der vier Grundkräfte, die wir kennen. Grundkraft heißt, wir können sie nicht (wie etwa den Magnetismus) auf eine noch grundlegendere Kraft zurück führen. Vielleicht gelingt das eines Tages einer Theorie der Quantengravitation, aber die haben wir noch nicht. Unsere Beschreibung der Effekte der Kraft ist hingegen sehr exakt.

    Du widerholst auch nur, was wissenschaftlich “feststeht” (wofür es sicher einen Grund gibt)

    Fest steht, dass die Gravitationstheorie nach der Allgemeinen Relativitätstheorie (die für geringe Geschwindigkeiten, kleine Raumzeitkrümmungen und unter Abwesenheit von signifikanten Scherspannungen und Drücken die Newtonsche Gravitationstheorie als Näherungslösung hat) die Natur im Rahmen unserer Messgenauigkeit perfekt beschreibt. Ich habe schon über Doppelpulsare geschrieben, an denen man gleich mehrere (5) Vorhersagen der ART überprüfen konnte. Und mit der Newtonschen Näherung navigierte die NASA die Sonde TESS neulich so um den Mond herum, dass die resultierende Bahn keinerlei Korrektur mehr bedurfte. Blöde ist nur, dass die ART an Singularitäten unsinnige Werte liefert.

    Soweit ich das sehe, lohnt es sich beim Thema Gravitation aber, wenn sich jeder abweichend vom sog. Mainstream seine Gedanken macht – unabhängig davon, ob er es kann oder nicht.

    Soweit ich das sehe, macht es keinen Sinn, sich Gedanken über Dinge zu machen, die man nicht vollständig versteht (um sie dann vielleicht im Selbststudium zu begreifen, schon, aber nicht, um diejenigen zu kritisieren, die sie verstanden haben; ich gehe ja auch nicht in eine Herzklinik oder das Entwicklungscenter von Ferrari und erkläre den Anwesenden, dass ich zwar keine Ahnung hätte, aber mir sicher sei, dass sie grundsätzlich was verkehrt machten). Aber glücklicherweise steht die Tür zur Wissenschaft jedem offen. Man kann eine Arbeit bei jedem Fachmagazin oder jeder Konferenz einreichen, zwar nicht ganz umsonst, aber man wird davon auch nicht arm (teuer sind in der Forschung vor allem die Gehälter der Forschenden und natürlich die Großgeräte, aber die muss man nicht unbedingt selbst einsetzen; Theoretiker arbeiten mit Papier und Bleistift).

    Kopiert man den Mond und platziert ihn um 180° versetzt auf dem selben Orbit des Vorhandenen, verlagert sich das Baryzentrum des gesamten Systems in den Erdmittelpunkt, was zur Folge hat, dass die Erde nun ruht. Auf die Erde wirkt nun zwei mal die Anziehungskraft der Körper in entgegengesetzte Richtung und hinzu kommt die Kraft, die zwischen den beiden Monden wirkt. Diese Kraft ist nun für die Beschleunigungen verantwortlich, mit denen sich die Monde um die Erde bewegen und man kann sicher sein, dass es andere (höhere) sind, als jene, mit welchen die Massen der Monde und der Erde bestimmt wurden.

    Dem widerspricht die Gravitationstheorie ja auch nicht. Der Mond ist aber auch sehr massiv gegenüber der Erde, 1,23% der Erdmasse bringt er auf. Wenn man einen zweiten Mond hinzufügt, steigt die Gesamtmasse des System um 1,23%, also wird sich die Umlaufgeschwindigkeit der Monde ein wenig erhöhen. Die Planeten, die die Sonne umkreisen, bringen es im Vergleich nur auf 1/500 der Sonnenmasse, 0,2%. In guter Näherung umkreist jeder Planet die Sonne so, als ob er alleine wäre. Wenn man es genauer berechnen will, kann man alle Kräfte aller Objekte aufeinander berechnen, aber dann braucht man einen Computer und kann die Lösung nur numerisch bestimmen. Aber das macht man und sagt damit die Bahnen der Planeten für Jahrhunderte oder Jahrtausende sehr genau voraus. Immer gemäß Newton.

    Dunkle Materie findet sich mMn deswegen genau in jenen Dingen, die Wissenschaftler als irrelevant betrachten.

    Dunkle Materie befindet sich jedenfalls nicht in der leuchtenden, denn sie ist gemäß der Rotationskurven von Galaxien und gemäß der Lichtablenkung von Hintergrundgalaxien anders im Raum verteilt, als die leuchtende Materie. Scheibenförmige Spiralgalaxien haben kugelförmige Halos aus Dunkler Materie.

    In Cavendish-Experimenten wird nämlich ausschließlich nur der Fall “Eine Erde und ein Mond” – also zwei Massen betrachtet.

    Im Sonnensystem aber nicht. Wenn man genau rechnet (Ephemeridenrechnung).

    Für umlaufende Systeme aber gilt allgemein Massen von Partikeln im Medium, z.B. Sterne in Galaxien oder Gipspulver, dass man gemächlich bis zur vollständigen Aushärtung in rotierendes Wasser schüttet.

    Das Bild habe ich jetzt wieder nicht verstanden.

    Je mehr Massen nun darauf Einfluss nehmen, desto höher wird die Rotationsgeschwindigkeit

    Schon klar, und wenn man eine Galaxie als Masse rechnet, dann beeinflusst die Verteilung der Masse über die Form der Galaxie die Bewegung eines Probeteilchens. Das einzelne Teilchen spielt dann keine Rolle mehr, aber die Menge der Teilchen schon.

    Theoretisch kann ich meine Thesen sogar berechnen, nur leider habe ich als Laie kaum oder gar keinen Zugang zu wissenschaftlichen Beobachtungen, z.B. finde ich keine Stelle im Netz, wo Galaxien auf ihre Dichte hin untersucht werden und deswegen bringe ich auch keine spruchreifen Formeln hervor

    Auf arXiv finden sich mit Sicherheit zig Papiere mit Rotationskurven von Galaxien. Alternativ kann ich den Gaia-DR2-Katalog empfehlen, da kann man kostenfrei auf die Bewegung von 7 Millionen Sternen der Milchstraße durch den Raum zugreifen. Es gibt Karten, wie die Geschwindigkeiten verteilt sind. Es gibt Rotationskarten der Magellanschen Wolken. Man kann die Bewegung der Sterne in offenen Sternhaufen analysieren, da stört auch die dünn verteilte Dunkle Materie fast gar nicht. Nur zu, ran ans Werk, Daten auswerten, Paper schreiben, einreichen, berühmt werden!

  60. #60 Nicht von Bedeutung
    27. Mai 2018

    Hallo Alderamin

    aber nicht, um diejenigen zu kritisieren, die sie verstanden haben

    Das Bild habe ich jetzt wieder nicht verstanden.

    Dieses fehlende Verständnis scheint genau der springende Punkt bei meiner Kritik an diejenigen zu sein, die meinen, sie hätten die Gravitation verstanden.
    Es fehlt nämlich nicht nur dir, sondern auch vielen anderen. Eine Galaxie ist um Einiges “fester” als ein Sonnensystem, aber lange nicht so “fest”, wie eine starre Scheibe. Das ist vergleichbar mit Gipspulver, welches man in klares Wasser rührt. Je mehr Gips hinzukommt, desto steifer wird die Masse und je nach Konsistenz ändert sich ihr Rotationsverhalten auf verschiedenen Radien. Im Gegensatz zu anderen Theorien, muss ich bei meiner weder Kepler-Kurven noch Dunkle Materie in Galaxien annehmen.
    Rotationskurven nützen mir leider recht wenig, weil ich genau diese mit meinen Formeln schon berechnen kann. Was mir fehlt sind tatsächliche Werte von Dichten sichtbarer Materie in Galaxien. Bei Arxiv.org habe ich auch schon ein Konto nur leider sind mir schon seit geraumer Zeit die Ideen ausgegangen, an ein Endorsement zu kommen.
    Um noch mal auf die Frage zurück zu kommen, wo ein Orbit aufhört und der nächste anfangt… Wie bestimmt man denn, Geschwindigkeitsverteilungen bei Maxwell-Boltzmann oder Frequenz- bzw. Wellenlängenverteilungen in Planckspektren? Durch Intervalle, nicht wahr? Genau das ist auch das probate Mittel bei meinen Berechnungen und die funktionieren in der Theorie nun mal recht gut. Deswegen unterscheidet sich mein Verständnis von Gravitation ja auch so unheimlich von gängigen Theorien.

  61. #61 Captain E.
    28. Mai 2018

    @Nicht von Bedeutung:

    Wenn jemand in einem Forum oder einem Blog-Kommentarboard aufschlägt und erzählen will, er habe dieses oder jenes erheblich besser verstanden als die Profis, die das teils seit Jahrzehnten machen und sich auf Erfahrungen etlicher Wissenschaftlergenerationen stützen, erwirbt er sich sehr schnell den Ruf eines Angebers und Spinners – und das zumeist zu Recht.

    Nehmen wir aber einmal an, du hättest tatsächlich etwas gefunden, was allen Astronomen und Physikern bislang einfach durchgerutscht sei. Bringt es dir etwas, das uns zu erzählen? Nein, sicherlich nicht. Du müsstest deine Ideen bei den Profis anbringen, und da gilt, was der Hausherr dir schon gesagt hat: Schreib es auf und veröffentlich es. Falls dir die Grundlagen dazu fehlen, sind deine Ideen leider schon rein formal nichts wert. Lerne, dich wissenschaftlich korrekt auszudrücken und bring deine Ideen unter das wissenschaftliche Volk. Aber rechne damit, dass du auf dem Weg dahin erkennen könntest, falsch gelegen zu haben.

  62. #62 Nicht von Bedeutung
    28. Mai 2018

    @Captain E.:
    Ich sage ja nicht, ich hätte Gravitation besser verstanden, sondern eben nur anders – das lässt keine der beiden Seiten als “Idioten” dastehen. Aber sich darüber zu äußern bringt wahrscheinlich weder etwas bei Wissenschaftlern noch bei Leuten, die nur Interesse bekunden, wobei denen selbst Grundlagen fehlen – mir fehlen Grundlagen jedenfalls nicht.
    Es geht dabei in erster Linie um Verständnis und dieses bringen mir höchstens die Leute bei Dissident Science entgegen. Nur selten gibt mal jemand (z.B. Alderamin) zu, etwas nicht verstanden zu haben, z.B. meinen Vergleich mit flachen umlaufenden Systemen und Gipsmasse.
    Sagen wir es mal so: Den Wissenschaftlern geht schon seit etwa 200 Jahren etwas durch, nur wollen sie es nicht wahr haben, weswegen sie meine Thesen ablehnen (fehlendes Endorsement bei ArXiv.org) und hinzu kommt das (all)gemeine Volk, welches eher Wissenschaftlern glaubt, als einem Spinner, wie mir, wobei es aber nicht bereit oder gar fähig ist, meine Thesen nachzuvollziehen. Solche Situationen, wie die Meine gab es in der Wissenschaft zur Genüge – Platon, Galilei, Kopernikus, Einstein, Zwicky… – und nur bei den beiden letzt genannten dauerte es nur wenige Dekaden, bis sie ernst genommen wurden, die Anderen haben ihren Erfolg selber gar nicht mehr miterlebt. Ich gehöre lieber zu der Truppe von Einstein und Zwicky, aber dank Peer-Review-Verfahren bleibt das wohl Wunschdenken.
    Es scheint so, als würden Wissenschaftler lieber sehen, wie ein Einzelner scheitert, als viele aus deren eigenen Reihen. Wen also sollte man wie erreichen, wenn einem keiner zuhört? Das Internet scheint mir da eine gute Platform zu sein.

  63. #63 Captain E.
    28. Mai 2018

    @Nicht von Bedeutung:

    Ach nein, bitte nicht schon wieder die Spinnerverteidigung! Ja, du hast recht, dass mancher genialer Mensch bei seinen Zeitgenossen als durchgeknallter Spinner gegolten hat. Einstein und Zwicky gehörten meines Wissens allerdings nicht dazu. Allerdings solltest du bedenken, dass die allermeisten Menschen, die ihren Zeitgenossen als durchgeknallte Spinner gegolten haben, tatsächlich auch durchgeknallte Spinner gewesen sind.

    Du behauptest, du hättest die Grundlagen, um einen neue phyikalische Theorie entwickeln zu können. Nun, das sehe ich bislang nicht, und deine weinerliche Haltung, man wolle dich mit aller Macht davon abhalten, dies zu tun, legt den Verdacht nahe, dass du einer der vielen ungezählten durchgeknallten Spinner mit galoppierender Paranoia bist. Der Effekt wurde sogar schon untersucht und hat einen Namen: Dunning-Kruger-Effekt. Damit wird die systematische fehlerhafte Neigung relativ inkompetenter Menschen bezeichnet, das eigene Können zu überschätzen und die Kompetenz anderer zu unterschätzen.

    So, und nun liegt es wirklich einzig und allein an dir, diesen Verdacht auszuräumen. Willst du ernst genommen werden, dann benimm dich auch so.

  64. #64 Nicht von Bedeutung
    28. Mai 2018

    @Captain E.
    Ich diskutiere nicht in dieser Form mit dir. Sei nicht so herablassend, ohne Argumente zu hinterfragen, sonst bleibt der Dunning-Kruger-Effekt nur an dir hängen.

  65. #65 Captain E.
    28. Mai 2018

    @Nicht von Bedeutung:

    Ein bis zwei letzte Tipps für dich:

    1. Das Internet ist mit Sicherheit nicht nicht der richtige Platz, um vermeintlich neue wissenschaftliche Hypothesen publik machen zu wollen.

    2. Dieser Blog ist schon gleich gar nicht die richtige Spielwiese für jemanden wie dich.

  66. #66 Nicht von Bedeutung
    28. Mai 2018

    @Captain E.:
    Nirgendwo gibt es eine Platform für die Veröffentlichung vermeintlich neuer Theorien, dafür hat die Wissenschaft mit ihrem Peer-Review gesorgt. Das Internet ist die Öffentlichkeit und dort lassen sich vermeintlich neue Theorien halt am besten bewerten, weil dort mehr kritische Denker vorhanden sind, als in Kreisen der Wissenschaft. Das Problem ist die Nachvollziehbarkeit, welche btw. in der Wissenschaft nicht gegeben ist. Mehr noch – Wissenschaftler verweigern die Nachvollziehbarkeit in der selben Form wie du, indem sie vermeintlich neue Theorien unreflektiert denunzieren und dabei machen sie nicht mal vor den eigenen Reihen halt (siehe Alexander Unzicker).
    Der letzte Tipp für dich (selbst in der Annahme, dass du ihn genauso wenig beherzigst, wie ich die Deinen):
    Sei nicht so herablassend.

  67. #67 Alderamin
    28. Mai 2018

    Der Peer-Review ist die Methode, mit der die Wissenschaftsgemeinschaft sich und das einreichende Individuum vor Selbsttäuschung schützt. Peer Reviews finden heraus, ob ordentlich gearbeitet, gemessen und begründet wurde. Nicht Thema des Reviews ist hingegen, ob das, was herauskommt, konform mit geltenden Theorien geht oder nicht – natürlich darf man jede Theorie auch angreifen, wenn man nachfollziehbare Argumente vorweisen kann. Der Peer Review ist die Qualitätssicherung der Wissenschaft und hat nachhaltig zum Erfolg der Wissenschaft beigetragen.

    Außerhalb von Peer Reviews kann man überall veröffentlichen, auf Webseiten, in Büchern, in Youtube-Videos etc. Das war noch nie so einfach wie heute. Dennoch würde ich jedem Laien raten, sich nur auf Quellen zu verlassen, die Peer-reviewt wurden (oder sich auf solche Quellen stützen). Wenn man einen Schaden am Auto hat und selbst keine entsprechenden Kenntnisse, sollte man ja auch zur Werkstatt gehen und sich nicht jemanden suchen, den man nicht kennt und der keine Fachausbildung vorweisen kann, nur weil er behauptet, er könne alles besser. In der Wissenschaft geht es nicht darum, Mehrheiten durch Werbung zu gewinnen. Es geht darum, die Natur nachprüfbar besser zu beschreiben als andere vorher. Wenn man das kann, schafft man jeden Peer Review und bringt die Wissenschaft voran. Wenn nicht, sollte man besser die Füße still halten.

  68. #68 Nicht von Bedeutung
    28. Mai 2018

    Hallo Aldemarin
    Dieses Facharbeiter-Argument ist ebenso stichhaltig, wie das Dunning-Kruger-Argument, nämlich gar nicht. Wenn ein Laie des Fensterputzerhandwerks sieht, dass ein Fensterputzer vom Fach nur eine Seite einer Scheibe putzt, hat er die verdammte Pflicht, dem Fachmann mitzuteilen, dass so eine Scheibe zwei Seiten hat.
    In Physik und Kosmologie geht es aber nun mal nicht um solche trivialen Dinge, sondern um weitaus tiefgreifendere – z.B. darum, dass Entdeckungen der Physikgeschichte in der falschen Reihenfolge erfolgten. Ganz sicher hätte Kepler sein drittes Gesetz von vorne herein auf Sonnensysteme begrenzt, wenn er vom Rotationsverhalten der meisten Galaxien gewusst hätte. Evtl. wäre er noch darauf gekommen, dass man dieses dritte Gesetz nur mit \frac{4\pi^2}{4\pi^2}=1 hätte erweitern brauchen, um es auf alle annähernd scheibenförmigen Rotationen auszuweiten. Die Vorlage für Newton wäre dann eine ganz Andere gewesen aber er wäre dennoch auf sein Gravitationsgesetz gekommen, nur halt mit einem anderen Term für G (welches bei Newton selber btw. niemals zur Debatte stand, sondern erst 1876 nach zahlreichen Cavendish-Experimenten eingeführt wurde): $latex G=\frac{v^2 r}{M}
    In Folge dessen hätte man evtl. ganz andere Experimente durchgeführt und dabei festgestellt, das G erstens vom Verhältnis und zweitens von der Anzahl beteiligter Massen abhängt.
    BTW. wurden weder Kepler 3 noch das Gravitationsgesetz so streng wie Theorien von heute oder überhaupt Peer-Reviewed und da viele meinen, sie hätten Gravitation hinreichend verstanden, werden sie das sicher auch nicht mehr, was den Nachteil hat, dass man Dunkle Materie einfach in Kauf nehmen muss – alles Andere sind die Idiotien irgendwelcher Spinner, wie mir. Peer-Reviewed das mal.

  69. #69 Uli Schoppe
    28. Mai 2018

    Nunja ohne dem Freund des Gallileo gambits das Wort reden zu wollen: das das System und der dämliche Veröffentlichungs- und Zitiertwerdenzwang so seine Tücken hat war ja auch auf scienceblogs.de schon Thema. Ist ja definitiv tauglich, aber ob da alles so bleiben kann wie es ist nur weil es einem definitiv auch die Spinner vom Laib hält?

  70. #70 Captain E.
    29. Mai 2018

    @Nicht von Bedeutung:

    Tja, bin ich herablassend? Ich denke, nein. Zumindest bin ich nicht so herablassend, von mir selber zu glauben, eine physikalische Hypothese entwickelt zu haben, die das wissenschaftliche Weltbild umstürzen könnte. Du tust dagegen genau das, und das hört sich für mich sehr stark nach einem galoppierenden Dunning-Kruger-Effekt an. Wenn ich richtig liege, bist du natürlich völlig berauscht von deiner nicht vorhandenen Kompetenz und somit völlig beratungsresistent. Ich werde also weitere Versuche, dich zu überzeugen, ab sofort einstellen.

    Ein paar letzte Anstöße für dich:

    1. Das Internet ist wirklich nicht der richtige Ort für dich, weil du im Internet nur Leute von Quatsch wie Verschwörungstheorien (Kennedy-Attentat, 9/11 usw.) überzeugen kannst. Willst du dich wirklich auf diese Stufe stellen? Da könntest du dich nämlich genauso gut auf die Seite der Chemtrailer schlagen. Dazu findest du auch jede Menge im Internet, was aber nichts daran ändert, das völliger Blödsinn ist.

    2. Du unterstellst den Physikern und Astronomen, sie hätten verstanden, was Gravitation ist. Dazu müssten sie aber auch verstanden haben, warum Gravitation den Raum krümmt. “Warum”-Fragen zu beantworten ist aber Sache der Naturwissenschaft nicht. Und bislang kann auch noch niemand so recht sagen, was dieser Raum eigentlich sein soll. Da wartet alle Welt auf die Entwicklung einer brauchbaren Theorie zur Quantengravitation. Alderamin hat dir bereits ein Beispiel genannt, das von einem Physiker stammte und schön demonstriert, dass sich der Mann der Unzulänglichkeiten nur zu gut bewusst ist.

    3. Du wirst den Physikern auch vor, etwas seit 200 Jahren zu übersehen, was du nun angeblich entdeckt haben willst. Das ist ja schon deshalb völliger Unsinn, weil der Begriff der Gravitation vor 200 Jahren noch ein ganz anderer gewesen ist. Damals war Newton noch State of the Art. Vor etwas mehr als 100 Jahren hat Einstein das Weltbild völlig umgeworfen. Womöglich noch revolutionärer war die Einführung der Quantenmechanik. Die Physik ist also nur zu bereit, ihr Weltbild völlig auf den Kopf zu stellen, wenn das neue besser funktioniert als das alte. Und solchen Menschen wirfst du allen Ernstes vor, etwas übersehen zu haben, was dir mal so eben aufgeallen sein soll? Schlimmer noch, sie sollten sich mit Händen und Füßen dagegen wehren? Entschuldigung, aber das ist Quatsch, und es zeugt von einer unerträglichen Arroganz deinerseits.

    4. Zum Letzten Mal: Wenn du wirklich auf etwas gestoßen sein solltest, dann schreib es auf und präsentier es der Fachwelt. Wenn du das nicht kannst, halt den Mund und lern, wie geht. Wenn du im Internet weiter so weitermachst, wirst du dich nur als durchgeknallter Spinner präsentieren. Falls deine Hypothese korrekt sein sollte, wird sie irgendwann jemand entdecken, der die Fachwelt zu überzeugen weiß. Der wäre dann aber garantiert nicht du, und niemand würde sich deiner als Entdecker erinnern.

  71. #71 Alderamin
    29. Mai 2018

    Die Verbesserungsvorschläge am Veröffentlichungsverfahren gehen aber nicht dahin, den Peer Review abzuschaffen, sondern ihn unabhängiger von den Magazinen (und damit weniger bestechlich) zu machen. Die Geldmacherei der Magazine ist das Hauptproblem – und bei manchen sollen ja sogar Reviews verkauft worden sein.
    Im großen und ganzen hat der Betrieb aber bisher funktioniert. Schwarze Schafe gibt‘s immer mal, gelegentlich.

  72. #72 Nicht von Bedeutung
    29. Mai 2018

    @Uli Schoppe:
    Ich meine nicht Recht zu haben, weil ich vom Establishment verunglimpft werde, sondern ganz allein deswegen, weil mein Vorgehen und daraus folgende Resultate und Folgerungen logisch sind – da kann kein Peer-Review etwas dran ändern, da hilft nur nüchternes Nachdenken über die “Grundlagen”.
    MMn hat das Peer-Review durchaus die gute Seite, dass es Spinner aus der Wissenschaft raus hält, nur leider auch den Nachteil, dass gute Ideen dadurch auch aus der Wissenschaft herausgehalten werden, weil man keinen zitieren kann. Wie ein durch Inzest degenerierter Gen-Pool verwaist deswegen der Wissens-Pool der Wissenschaft durch strenges Peer-Review. Und solange an diesen fragwürdigen Dingen, wie Raumzeitkrümmung, Dunkle Energie und Dunkle Materie festgehalten werden soll, hat gnadenlose Logik in der Wissenschaft keinerlei Chance.

  73. #73 Nicht von Bedeutung
    29. Mai 2018

    Hey Captain E.:
    Zum letzten mal: Ich diskutiere mit dir nicht auf diesem Level.
    1. Hättest du meine Kommentare hier gelesen, dann wüsstest du, dass ich schon seit längerem versuche, meine Ideen einer Fachwelt zu präsentieren, diese aber nicht gewillt ist, diese auch nur geringfügig in Erwägung zu ziehen, weil sie keinen Urknall, keine Raumzeitkrümmung und was sehr viel wichtiger ist keine Dunkle Energie und Dunkle Materie enthält.
    2. Meine Hypothesen sind längst keine Hypothesen mehr, sondern tatsächlich eine Theorie mit allem was dazugehört.
    3. Glaube er bloß nicht, dass ich ohne Erlaubnis hier etwas veröffentlichen oder verlinken werde.
    4. Kannst du mir überhaupt folgen oder verstehst du die Grundlagen nicht?

  74. #74 stone1
    29. Mai 2018

    War ja nur eine Frage der Zeit, bis es auch hier passiert. Urknall, ART, DM und DE beim Gips anrühren vom Tisch gewischt. Es wundert mich nur, dass anscheinend nie jemand mal mit kleineren Brötchen daherkommt, nein es muss immer gleich die große Torte sein.

    Wenn ein Laie des Fensterputzerhandwerks sieht, dass ein Fensterputzer vom Fach nur eine Seite einer Scheibe putzt, hat er die verdammte Pflicht, dem Fachmann mitzuteilen, dass so eine Scheibe zwei Seiten hat.

    Der Fensterputzer wird sich sicher sehr über diesen wertvollen Hinweis von einem beflissenen Laien der Fensterreinigung freuen und gleich beim nächsten Mal versuchen, die Innen- und Außenseite des Fensters gleichzeitig zu putzen.

    Duckundwech…

  75. #75 Captain E.
    29. Mai 2018

    @stone1:

    Echt, dafür von meiner Seite aus Daumen hoch für eine gelungene Fortführung eines völlig schiefen Bildes. Jetzt hast du es gerade gerückt, und dafür vielen Dank und herzlichen Glückwunsch! 🙂

  76. #76 Alderamin
    29. Mai 2018

    weil sie keinen Urknall, keine Raumzeitkrümmung und was sehr viel wichtiger ist keine Dunkle Energie und Dunkle Materie enthält.

    Keinen Urknall? Dann ist sie definitiv falsch, der ist nun wirklich hinreichend belegt, vom Mengenverhältnis der Elemente über die Hintergrundstrahlung, das Alter der ältesten Sterne, die zunehmende Metallizität der Sternenpopulationen bis zur Expansion des Universums. Wir blicken bis an die Grenze der Entstehung der ersten Galaxien und können wegen der Lichtlaufzeit gewissermaßen zuschauen, wie das Weltall entstanden ist.

    Florian hat da neulich schon einen Artikel zu geschrieben: die Wissenschaft ist ein Gerüst von Theorien, die sich gegenseitig stützen. Wenn man da an einer Stelle dreht, dann hat das riesige Konsequenzen, die sich überall zeigen würden. Und es ist ja nicht so, dass man sich das alles nur ausgedacht hat – eine Theorie ist erst eine Theorie, wenn sie Beobachtungen erklärt und mit Messungen bestätigt werden kann (bzw. wenn ihre Falsifizierung durch Messungen nicht gelingt). Theorien können sich ändern – die Messdaten bleiben.

    2. Meine Hypothesen sind längst keine Hypothesen mehr, sondern tatsächlich eine Theorie mit allem was dazugehört.

    Ach, wer hat denn je versucht, sie experimentell zu falsifizieren? Wenn niemand sie in Erwägung zieht?

    Wenn Du mal einen echten Rebellen der Kosmologie kennenlernen willst, dann lies oben die Kommentare von Oliver Müller, der hat das Titelblatt der Zeitschrift Science erobert, obwohl er mit der Dunklen Materie über Kreuz liegt. Du siehst, man kann sich mit alternativen Theorien Gehör verschaffen, wenn man gute Argumente hat. “Alles was Millionen Wissenschaftler seit 200 Jahren machen ist falsch” ist kein gutes Argument. Das gibt unweigerlich einen wissenschaftlichen Bauchplatscher. Vom Zehner.

  77. #77 Nicht von Bedeutung
    29. Mai 2018

    @stone1:
    Wenn man genau hinsehen würde, würde man feststellen, dass die Gravitationskonstante bereits 1687 Geschichte gewesen wär, wenn der Verlauf der Physik-Geschichte ein Anderer gewesen wär. Ohne die Gravitationskonstante gäbe es kein Kappa in den Einsteinschen Feldgleichungen, welche die Grundlage der ART bilden. Ohne Kappa bleibt von der ART nichts weiter übrig, als ein Haufen ordinärer Differentialgleichungen, die irgendwas beschreiben, nur nicht den Umstand, dass 2kg die Raumzeit nicht wirklich doppelt so tief eindellen, wie 1kg. Wer weiß, ob es ohne die Gravitationskonstante jemals zu einer ART gekommen wäre. DM und DE wären ohne diese jedenfalls vom Tisch und ohne DE gab es auch keinen Urknall. Das alles ist aber unerheblich, solange es ein “Laie” sagt, der sich mit den Grundlagen nicht beschäftigt haben soll.
    Dein Duckundwech… war durchaus berechtigt, denn von gleichzeitig hat ja keiner etwas gesagt. Höchstens von Fenster öffnen und die Rückseite putzen, statt stundenlang, in der Hoffnung irgendwann mal durchsehen zu können, den längst nicht mehr vorhandenen Staub von der Vorderseite wischen zu wollen. Das ist eine Metapher für Sichtweisen – falls dir das Wort Metapher überhaupt etwas sagt.
    Duckundwech…

  78. #78 rolak
    29. Mai 2018

    falls dir das Wort Metapher überhaupt etwas sagt

    Ja aber sicher sagt uns das etwas – immerhin ist seine Bedeutung am 4.7.11 endgültigt fixiert worden (©Gernhardt):

    Herr Kapitän, der Steuermann
    hat grade lallend kundgetan,
    er brächte jetzt das Schiff zum Sinken –
    Metapher wirklich nicht mehr trinken.

    Übrigens nachträglich beste Gückwünsche zur Wahl des nicks – ‘Nicht von Bedeutung’ beschreibt Deine Texte perfekt. Grandios! Wie zum Beispiel bei Deinem Fensterputzer. Da war wohl der lokale Metaphern-Stimmigkeits-Kontrolleur in seinem Jahresurlaub, ne?

  79. #79 Nicht von Bedeutung
    29. Mai 2018

    @rolak:
    :rofl: Klar, von mir aus, war es bei meinem Metaphern-Stimmigkeits-Kontrolleur nur ein Jahresurlaub. Ändert aber leider gar nichts daran, dass die Geister der Wissenschaft bereits mindestens mehrere Dekaden im Ruhestand sind – ob das nun für dich (euch) von Bedeutung ist, oder nicht.
    Übrigens gilt für dich ab sofort auch dass, was für Captain E. bereits gilt – ich diskutiere nicht auf solch niedrigem Niveau. Es macht also auch für dich keinen Sinn mehr, mittel- oder unmittelbar auf meine Beiträge einzugehen, ich bin mir aber sicher, dass du es trotzdem tust, weil das letzte Wort hat ja der Mainstream. :rofl:

  80. #80 Nicht von Bedeutung
    29. Mai 2018

    Allen Anderen sage ich es mal so:
    Ich habe hier (mal wieder) eine Idee angeregt, die erstmal an Bedeutung gewinnen muss. Das schafft man aber sicher nicht, indem man meint das Ein oder Andere verstanden zu haben, während man Essentielles nicht verstanden hat. Kurz gesagt, man muss bereit sein, seine Sichtweise zu ändern und nicht bloß beteuern, man sei es. Zu meiner Sichtweise ist damit alles gesagt und ich werde mich dazu nicht mehr äußern. Über Möglichkeiten zu Veröffentlichungen (außer Peer-Review) kann man sich jedoch noch unterhalten und zwar ohne gegenseitige Veralberungen, wenns denn geht.

  81. #81 Captain E.
    29. Mai 2018

    @Alderamin & rolak & stone1:

    Habt ihr es gesehen? Da war jetzt der “Mainstream”. Der musste ja auch einfach noch kommen.

  82. #82 rolak
    29. Mai 2018

    gesehen?

    Schon, Captain, Sir, doch das kann nicht mir gegolten haben, fließt hier doch in Sichtweite der Rhein und nicht etwa der Main.

  83. #83 stone1
    29. Mai 2018

    @Capitain E.

    Da war jetzt der “Mainstream”

    Au contraire mon capitan! Der Ausguck meldet: es handelt sich hierbei eindeutig um die Donau, und jetzt geh ich wieder Brett spielen ähm Deck schrubben, da mir ja nicht mal zugetraut wird, das Wort ‘Metapher’ zu kennen, Aye.

    Den (c)Gernhardt fand ich trotzdem zum prusten köstlich, @rolak. Gut dass zu dem Zeitpunkt der Kaffee noch nicht fertig war. OT Ende.

  84. #84 Nicht von Bedeutung
    29. Mai 2018

    Nichts wird sich auf Erden ändern, bevor nicht alle gorisch werden. 😀

  85. #85 Captain E.
    29. Mai 2018

    @rolak:

    Komisch, bei mir gerade auch! Aber ob Rhein oder Main, am Ende fließt doch alles in die Nordsee.

  86. #86 Alderamin
    29. Mai 2018

    @Captain

    Die Donau aber nicht. Ist folglich nicht Mainstream 😀

  87. #87 Nicht von Bedeutung
    29. Mai 2018

    Her Ober, bring mir einen Kibel, mir wird von diesem Nonsens ibel. 😀

  88. #88 Captain E.
    29. Mai 2018

    @Alderamin:

    Die Donau aber nicht. Ist folglich nicht Mainstream

    Ja, die ist noch nicht einmal R(h)einkultur – selbstverständlich mit Ausnahme der schönen schwarzen Donau. 😉

  89. #89 Oliver Müller
    31. Mai 2018

    Ich will hier nur anmerken, dass ich solche “geniale Weltformeln” fast täglich in meinem E-Mail Briefkasten habe. Jeder Laie meint, er könne die Physik revolutionieren – aber der Mainstream unterdrückt diese. Inwischenzeit ist meine Standardantwort, dass ich die vorhergehenden E-Mails als Antwort schicke. 🙂

    Wer den Zusammenhang zwischen Rotationskurven, Dichte und Masse nicht versteht, hat schon mal nichts von Galaxien verstanden. Da hat der Captain wohl nicht ganz unrecht mit dem Dunning-Kruger-Effekt.

    Vor ein paar Tagen hatte ich sogar jemanden – unangemeldet – in meinem Büro stehen, der mir von Verschwörung bei der Zeitmessung vorschwafelte. Er log mir vor, dass es keine Angabe im Internet zu Millisekunden eines siderischen Tages gibt. Hat wohl nicht verstanden, was 5 Kommastellen nach der Sekunde bedeuten…

    Aber um konstruktiv zu bleiben: Für Laien-Wissenschaftler gibt es vixra.org, dort darf jeder seine verrückte Idee als Fachartikel publizieren. Ob sie ernst genommen wird, ist eine andere Frage. Eine kurze Google Suche hätte wohl weniger Zeit gekostet, als hier über das Peer-Review System zu jammern… Aber Kurzsicht ist wohl ein Symptom gewisser Leute… Mit System…

    @Nicht von Bedeutung
    Das ist jetzt keine Aufforderung für eine Diskussion, da ich sie sowieso nicht Lesen werde.

  90. #90 Uli Schoppe
    31. Mai 2018

    Lieber Oliver,
    ich kann ja verstehen das einem die Leute auf den Sack gehen. Ich empfehle aber einen anderen Umgang, wenn man gleich crank schreit setzt man sich für viele sofort ins Unrecht.
    Wenn mir einer vorrechnen will warum man mit einer Saturn nicht zu Mond kommt ist finde den Fehler für mich leicht. Da kenne ich mich aus.
    Hier weiss ich bis jetzt nicht wovon er redet, da wäre zumindest ein link auf eine ähnliche Diskussion wo das behandelt wurde sinnvoll.
    Man muss sich das Bild für Unbeteiligte anschauen: Er trifft viele schlau klingende Aussagen und die Antwort ist du bist doof. Und da wundert es einen das man dem zuhört? So funktioniert die Vermittlung von Wissen auf keinen Fall.

  91. #91 Nicht von Bedeutung
    31. Mai 2018

    +Uli Schope:
    Genau so siehts aus. Wenn sich zwei “Professionelle” (um Mainstreamer zu vermeiden) um DM oder MoND streiten und ein Laie hinzukommt, der einfach nur behauptet, dass einiges Vieles anders verlaufen wäre, wenn 1964 bereits so um 1600 stattgefunden hätte, gehen die “Professionellen” natürlich sofort auf den Laien los, weil sie ja wirklich alles verstanden haben. Nur leider versteht Alderamin z.B. mein Gleichnis mit Gips und Wasser nicht. Ich hingegen verstehe nicht, wieso man in Galaxien überhaupt Kepler-Kurven annehmen kann, wo man doch deutlich sieht, dass Rotationskurven von der Dichte rotierender Systeme abhängen. Wem glauben diese kurzsichtigen Mainstreamer eigentlich selbst auf den Sack gehen zu dürfen, nur weil jeder seit Dekaden den selben Bockmist studiert und nicht mal auf die einfachsten Dinge kommt?

    +Oliver Müller:
    Klar Google… eignet sich perfekt dazu um die eigenen Ideen zu finden. Habt ihr 0815-Mainstream-Lemminge eigentlich noch etwas anderes zu bieten als dieses nervige Papa oohm Ma ma? :rofl:
    Ach schiet… du gehörst ja nicht zum Mainstream. Oh wie traurig.

  92. #92 Nicht von Bedeutung
    31. Mai 2018

    Mit anderen Worten: Diskussionen lohnen sich ohnehin nicht, solange jeder von seiner eigenen Wahrheit überzeugt ist.

  93. #93 Alderamin
    31. Mai 2018

    @Uli Schoppe

    Würdest Du denn auch mit Flacherdlern diskutieren?

    Wer heutzutage noch Zweifel am Urknall hegt, der befindet sich auf Flacherdlerniveau. Da erübrigt sich jegliche Diskussion.

  94. #94 Alderamin
    31. Mai 2018

    @Nicht von Bedeutung

    Ich hingegen verstehe nicht, wieso man in Galaxien überhaupt Kepler-Kurven annehmen kann,

    In #55 hatte ich schon gesagt, dass Keplerkurven nicht vorliegen, wenn ein Objekt innerhalb der Milchstraße seinen Abstand zum Massenschwerpunkt ändert, weil sich dann die effektiv wirkende (im Radius eingeschlossene) Masse ändert. Die einzige mögliche Keplerbahn ist die Kreisbahn, denn eine Ellipsen-, Parabel oder Hyperbelbahn ändert den Abstand zum Massenschwerpunkt.

    wo man doch deutlich sieht, dass Rotationskurven von der Dichte rotierender Systeme abhängen.

    Du kannst Deine Hypothese auch nicht mit den Rotationskurven belegen, auf die Du sie gefittet hast (von wegen “sieht man doch”), Du musst eine Gleichung für die Gravitation aufstellen, daraus eine Vorhersage ableiten und die dann durch Beobachtungen an anderer Stelle belegen (z.B. Objekte mit verschiedenen “Belegungsgraden” der Orbits finden).

    kurzsichtigen Mainstreamer […] weil jeder seit Dekaden den selben Bockmist studiert […] ihr 0815-Mainstream-Lemminge

    Unterlasse bitte die Provokationen, sonst bist Du hier raus.

  95. #95 Nicht von Bedeutung
    31. Mai 2018

    Wie gesagt… Ich halte Diskussionen mit euch eh schon für sinnlos – nicht nur weil ich den Urknall ablehne, sondern auch, wei ein Objekt alleine gar keine Rotationskurve gemäß Kepler hervorbringen kann. Für Rotationskurven braucht man schon mehrere Objekte auf mehreren Radien zum Zentrum. Und mit der Form hat das noch viel weniger zu tun, erschwert die Sache nur. Selbst wenn seine Aussage noch irgend einen anderen Sinn haben sollte, der irgendwie mit der ART zusammenhängt, erklärt das immer noch nicht, warum man trotz ART noch Dunkle Materie überhaupt und Schwarze Löcher in galaktischen Zentren benötigt.
    Ich weiß nicht, woher du die Info hast, ich hätte meine Rotationskurven auf irgendetwas gefittet… kann ich nämlich gar nicht mangels konkreten Daten. Dass ich eine Gleichung für die Gravitation aufstellen muss, halte ich für ein Gerücht, denn das einzige, was sich am aktuellen Gravitationsgetezt ändert ist die Definition von G, welche nicht mehr konstant sein kann, womit für Kappa btw. selbiges gilt.
    Und was das Provozieren angeht… Ihr Mainstreamer provoziert mal wieder zu erst (siehe Beitrag #90 von Uli Schope). Und bevor ich mich von euch 0815-Mainstream-Lemmingen weiter zum Idioten machen lasse, provoziere ich lieber selbst. Schöne Zeit noch.

  96. #96 Alderamin
    1. Juni 2018

    @Nicht von Bedeutung

    Für Rotationskurven braucht man schon mehrere Objekte auf mehreren Radien zum Zentrum.

    Da wäre ich jetzt gar nicht drauf gekommen… wurde je was anderes behauptet?

    Selbst wenn seine Aussage noch irgend einen anderen Sinn haben sollte, der irgendwie mit der ART zusammenhängt, erklärt das immer noch nicht, warum man trotz ART noch Dunkle Materie überhaupt und Schwarze Löcher in galaktischen Zentren benötigt.

    Man braucht Dunkle Materie, weil die sichtbare Materie in Kombination mit Newton eine andere Rotationskurve ergibt, als beobachtet wird.
    Und man braucht kein Schwarzes Loch im Zentrum der Milchstraße, um die Rotationskurve zu beschreiben. Das 2000tel der Milchstraßenmasse spielt keine Rolle. Es ist aber da, die Sterne in seiner unmittelbaren Umgebung sieht man mit irsinnigen Geschwindigkeiten darum kreisen.

    Ich weiß nicht, woher du die Info hast, ich hätte meine Rotationskurven auf irgendetwas gefittet… kann ich nämlich gar nicht mangels konkreten Daten.

    Du sagtest, Du hättest eine waschechte Theorie. Das mindeste an einer Theorie ist, dass sie eine Beobachtung beschreibt. Ohne Datensatz hast Du keine Beobachtung. Du hast gar nichts. Du hast also nur irgendeine ausgedachte Formel, von deren Richtigkeit Du überzeugt bist, ohne einen einzigen Realitätsabgleich. Kein Wunder, dass die Profis Dich nicht ernst nehmen. In der Wissenschaft werden Theorien immer an Beobachtungen abgeglichen.

    Dass ich eine Gleichung für die Gravitation aufstellen muss, halte ich für ein Gerücht, denn das einzige, was sich am aktuellen Gravitationsgetezt ändert ist die Definition von G, welche nicht mehr konstant sein kann, womit für Kappa btw. selbiges gilt.

    Wenn G Deiner Meinung nach nicht konstant ist, musst Du eine Formel angeben, wie G sich mit welchem Parameter ändert und das anhand von Beobachtungen belegen.

    Ihr Mainstreamer provoziert mal wieder zu erst (siehe Beitrag #90 von Uli Schope)

    Er hat Dich nicht provoziert, er hat Dich verteidigt. Nicht mal das bekommst Du mit.

    Und bevor ich mich von euch 0815-Mainstream-Lemmingen weiter zum Idioten machen lasse, provoziere ich lieber selbst. Schöne Zeit noch.

    Ok, Du bist hier raus. Tschüß.

  97. #97 Uli Schoppe
    1. Juni 2018

    @Alderamin danke für die Mühe. Und ja, ich würde auch einem Flacherdler antworten. Da weis ich wie man argumentiert. Jedenfalls wenn ich meine er glaubt das wirklich und trollt nicht nur *g*

  98. #98 Captain E.
    1. Juni 2018

    @Oliver Müller:

    […]

    Wer den Zusammenhang zwischen Rotationskurven, Dichte und Masse nicht versteht, hat schon mal nichts von Galaxien verstanden. Da hat der Captain wohl nicht ganz unrecht mit dem Dunning-Kruger-Effekt.

    Mein eigenes Verständnis dazu bewegt sich natürlich auch nur auf niedrigem Niveau – nicht dass wir uns da vertun.

    Ich stehe bei solchen Diskussionen ja immer auf dem Punkt: Wer nicht in der Lage ist, eine Mehrheit der Fachleute oder meinetwegen auch nur eine einigermaßen große Minderheit zu überzeugen, der sollte sich fragen, ob er wirklich eine neue Idee am Wickel hat. Spätestens bei “Mainstream”, “Unterdrückung” und “Verschwörung” klingen bei mir alle Alarmglocken.

    Wobei natürlich niemand gesagt hat, dass es immer leicht sei, neue Theorien am Markt zu platzieren. Neue Theorien widersprechen akzeptierten alten oder erweitern sie, und das stört schon einmal den einen oder anderen älteren akzeptierten und respektierte Wissenschaftler. Wenn es dann noch die eine oder andere Verbindung gibt, also etwa Menschen noch leben, die an der alten Theorie mitgearbeitet hatten, wird die Sache noch einmal etwas schwerer.

    Aber Genialität hin oder her – wenn eine Idee in der Luft liegt, wird sie sich irgendwann durchsetzen, weil dann ja auch andere daran arbeiten werden, teils eben auch völlig unabhängig voneinander.

    Aber zumindest braucht sich niemand zu bemühen, mich persönlich von einer neuen physikalischen Hyopthese überzeugen zu wollen. Ob ich sie glaube oder nicht, ist völlig irrelevant, und daher glaube ich nichts neues, was nicht zuvor schon in der Fachwelt lang und breit diskutiert worden ist.

    Vor ein paar Tagen hatte ich sogar jemanden – unangemeldet – in meinem Büro stehen, der mir von Verschwörung bei der Zeitmessung vorschwafelte. Er log mir vor, dass es keine Angabe im Internet zu Millisekunden eines siderischen Tages gibt. Hat wohl nicht verstanden, was 5 Kommastellen nach der Sekunde bedeuten…

    Mal eine oft gemachte Beobachtung: Wer gerne und überall Verschwörungen wittert, zitiert gerne vermeintliche Fakten, die bei Licht betrachtet völliger Quatsch sind. Hier ein paar Beispiele:

    Bei 9/11 wären keine Juden gestorben, weil die Bescheid gewusst hätten und nicht zur Arbeit gegangen seien. Stimmt nicht, denn es sind jüdische Angestellte der Firmen in den Türmen gestorben. Manche haben den Anschlag natürlich dadurch überlebt, dass sie nicht in den Gebäuden waren, aber es war noch ziemlich früh, und auch einige Atheisten und Anhänger anderer Religionen waren noch nicht angekommen.

    Es sei noch nie ein Apollo-Astronaut an Krebs gestorben (im Hinterkopf die “hohe” Strahlung des van Allen-Gürtels oder auch Sonnenwind/kosmische Strahlung). Nun, zumindest Alan Shepard von Apollo 14 ist an Leukämie gestorben, und es sei ohnehin dahin gestellt, wie sich das persönliche Risiko verändert haben könnte. Die Doppelflieger John Young und Eugene Cernan sind beide tot, aber doch recht alt geworden, während der dritte, Jim Lovell, sogar noch am Leben ist.

    Der Kennedy-Attentäter Lee Harvey Oswald hätte keine militärische Ausbildung gehabt und hätte daher über mehrere hundert Meter hinweg sein Ziel gar nicht treffen können. Nun, zum einen kann man so etwas auch als Zivilist trainieren, und zum anderen war der Mann ein US Marine mit leicht überdurchschnittlichen Schießergebnissen.

    Und auch eine mutwillige Verdrehung der Tatsachen ist es (ich hatte es bereits erwähnt), wenn so ein selbst ernanntes Genie der Fachwelt unterstellt, dass die Professoren seit Jahrzehnten oder Jahrhunderten wider besseres Wissens an irgendwelchem Unsinn festhielten und den Studenten jedwedes kreative Denken austrieben. Derjenige muss so etwas wie Quantenmechanik, Relativitätstheorie, photoelektrischen Effekt, subatomare Teilchen, Neutrinos, Gravitationswellen, Urknall, expandierendes Universum oder auch nur “simple” Galaxien übersehen haben, für falsch halten oder viel zu früh datieren.

    Aber um konstruktiv zu bleiben: Für Laien-Wissenschaftler gibt es vixra.org, dort darf jeder seine verrückte Idee als Fachartikel publizieren. Ob sie ernst genommen wird, ist eine andere Frage. Eine kurze Google Suche hätte wohl weniger Zeit gekostet, als hier über das Peer-Review System zu jammern… Aber Kurzsicht ist wohl ein Symptom gewisser Leute… Mit System…

    Ich schätze, wer daran glauben will, dass seine Meinung unterdrückt wird, hält dieses Portal wohl auch nur für den Versuch, verkannte Genies als Quell der Belustigung zu missbrauchen.

  99. #99 rolak
    1. Juni 2018

    Versuch, verkannte Genies als Quell der Belustigung zu missbrauchen

    Bereits besungen worden, Captain.

  100. #100 Oliver Müller
    1. Juni 2018

    Was ich einfach nicht verstehe ist, dass es so viele Leute gibt, die *keinen* wissenschaftlichen/physikalischen Hintergrund haben, aber die Physik revolutionieren wollen. Sie behaupten zwar, die gängige Lehre sei falsch, verstehen diese aber gar nicht. Und dann sehen sie sich als Leute wie Einstein und Zwicky, ergo als Underdogs, wobei der Unterschied darin besteht, dass Einstein, Zwicky und co. ein riesiges wissenschaftliches Know-How hatten und vorallem ständig an ihren Ideen zweifelten. Diese Selbstreflektion vermisst man bei vielen dieser Laien. Bei Kritik wird dann sofort zurückgeschossen. Das hat Ähnlichkeiten zum Extremismus.

    @Uli

    wenn man gleich crank schreit setzt man sich für viele sofort ins Unrecht.

    Vielleicht war mein Ton scharf, aber ich habe “Nicht von Bedeutung” nirgends direkt beleidigt. In die andere Richtung sieht es mmn anderst aus.
    Ich sehe aber ein, dass es für Leute hier im Forum nicht trivial ist, Rotationskurven mit Massen zu verbinden und ich entweder mehr Hintergrund oder einen Link hätte geben sollen.
    Manchmal muss man aber Verschwörungstheoretiker beim Namen nennen. Ich finde es dabei lustig, dass gerade ich als Mainstream-Lemming bezeichnet werde und nicht als Verschwörungstheoretiker. 🙂
    Aber ich nehme deinen Kommentar zur Kenntnis und versuche mich in dieser Hinsicht zu bessern. 🙂

    @ Captain E
    Auf Youtube geistert sogar ein Video (mit über 30’000 Views) eines Verschwörungstheoretikers herum, der mich persönlich 20 Minuten lang als Mainstream Lügner bezeichnet. Er legt mir dabei Argumente in den Mund, die ich nie gesagt/geschrieben habe, und widerlegt dann diese um seine Alternativtheorie zu beweisen.

    Vielleicht habe ich deswegen keinen Nerv mehr für solche Leute.

  101. #101 Captain E.
    1. Juni 2018

    @Oliver Müller:

    Du, ein Mainstream-Lemming? Da lachen ja die Hühner! Oder sonstwer…

    Ich erinnere nur daran, dass du auch schon einmal kommentiert hast, dich ein wenig verfolgt zu fühlen. Im Gegensatz zu all diesen missverstandenen Pseudogenies hast du die nötige Ausbildung, was dich wahrscheinlich für viele bereits zum “Lemming” macht. Aber andererseits hast du einen Peer-reviewed Artikel platzieren können, den sich die Kollegen nun vornehmen können, der offenkundig so gar nicht “Mainstream” ist.

    Nun ja, dein Problem ist wohl, dass du für die Richtigkeit einer Hypothese streitest, die von der Mehrheit der zeitgenössischen Physikern als “unbrauchbar” abgetan wurde, und da stelle ich es mir durchaus schwierig vor, das Thema wieder auf den Tisch zu bringen.

  102. #102 Oliver Müller
    1. Juni 2018

    @Captain E
    🙂

    Ich erinnere nur daran, dass du auch schon einmal kommentiert hast, dich ein wenig verfolgt zu fühlen.

    Ja, und das hat auch seinen Grund in meiner persönlichen Erfahrung, hier ein Beispiel: Ich wurde von der DPG (Deutsche Physikalische Gesellschaft) eingeladen, einen Artikel über das Plane-of-satellites Problem zu schreiben. Einen Monat später kam dann die Ausladung, da dieses Phänomen gar kein Problem sei und es somit nicht interessant wäre, darüber zu schreiben. Meiner Meinung nach muss da innerhalb dieses Monats sich jemand aus dem Dunkle Materie Lager scharf dagegen ausgesprochen haben – natürlich ist dies nur eine Vermutung – aber was sonst könnte der Beweggrund sein? Zuerst eine Einladung aussprechen und diese mit einer solchen Begründung zu revidieren ist nicht die feine Art. In einem solchen Artikel hätte ich ja genau darstellen können, warum solche Strukturen ein Problem für das heutige Modell sind. Aber einfach zu behaupten, dass es kein Problem ist, ist ein Hohn gegenüber der ganzen Fachliteratur, die sich inzwischenzeit recht einig ist, dass da etwas seltsames vorgeht. Ich hoffe, dass erklärt meine Grundparanoia. 🙂

    Warum es ein Problem für die Kosmologie ist, habe ich übrigens nun in einem Blogartikel erläutert: https://prosaderphysik.wordpress.com/2018/04/07/das-plane-of-satellites-problem/

  103. #103 Captain E.
    1. Juni 2018

    Der andere Grund wäre, dass einer sich die lästige Konkurrenz vom Hals schaffen wollte. Wenn man den “aufmüpfigen Jungspund” auslädt, kann man in Ruhe selber einen Vortrag/Artikel verfassen – das ist aber auch nur so eine Vermutung.

  104. #104 Oliver Müller
    1. Juni 2018

    @ Captain E.

    Der andere Grund wäre, dass einer sich die lästige Konkurrenz vom Hals schaffen wollte. Wenn man den “aufmüpfigen Jungspund” auslädt, kann man in Ruhe selber einen Vortrag/Artikel verfassen – das ist aber auch nur so eine Vermutung.

    Guter Punkt, welcher aber schlussendlich in die gleiche Richtung geht und der Hauptpunkt meines Artikels war: Wissenschaftler sind Menschen und somit der Soziologie unterworfen. 🙂

    Zur ganzen Diskussion über die Verschwörungstheoretiker, Scharlatene, und Astrologen habe ich heute meine Frust von der Seele geschrieben:
    https://prosaderphysik.wordpress.com/2018/06/01/die-schattenseiten-der-astronomie-astrologen-verschwoerungstheoretiker-und-scharlatane/

  105. #105 Uli Schoppe
    1. Juni 2018

    @Oliver spannend das dich Astrollogen als sowas wie ein Mitglied der Gemeinde sehen. Die sind schräg, weil jemand einer alternativen Theorie die etwa genau so viele Probleme hat wie die bevorzugte Theorie imho eine Stimme gibt ist er möglicherweise ein “Guter”. Strange ^^ Das Lügner Video tue ich mir bei Gelegenheit mal an.

  106. […] zwischen 500 Milliarden und 3 Billionen Sonnenmassen. Wie man Galaxien wiegt, haben wir schon bei NGC1052-DF2 gelernt – man betrachtet die Geschwindigkeiten der Kugelsternhaufen, welche die Galaxie umkreisen. Im […]

  107. #107 Uli Schoppe
    3. Juni 2018

    Ich setze meine Frage mal hier rein. Bitte nicht vergessen, ich bin der der in seiner Abi Vorklausur das Pauli-Prinzip komplett ignoriert hat aus unerfindlichen Gründen und entsprechend belohnt wurde. Kann sein das ich gerade an etwas total Elementarem vorbei gucke.
    Wie bildet dark matter Strukturen? Die Gravitation wirkt, potentzielle Energie nimmt ab und kinetische Energie nimmt zu. Die muss doch zum Verklumpen irgendwie weg. Im Detektierbaren kann das nicht sein per Definition. Wie macht die das?

  108. #108 Oliver Müller
    3. Juni 2018

    @Uli
    Eine sehr gute Frage. Ich könnte ein Artikel dazu schreiben (eigentlich eine gute Idee). Aber was du sagst, stimmt schon im Prinzip. Dunkle Materie verdichtet sich vorallem durch Gravitation. Sie zieht sich gegenseitig an, und es entstehen zuerst sogenannte sheets, dann Filamente, und dann knotet sich die Materie ab. Dies nennt man den Zeldovich Kollaps. Darum hat es in den Bildern zu Dunkle Materie Simulationen immer solche Filamente und Löcher, welche man auch auf grossen Skalen in Universum sieht, z.B. hier https://www.sdss3.org/science/gallery_sdss_pie2.php.
    Dies ist zwar eine Karte echter Galaxien, aber da die Galaxien in den Dunkle Materie Potentialen sitzen (nach der Standardtheorie), ist dies auch eine Karte der Dunklen Materie. Diest ist jedoch nur die Erklärung für die “grossen Strukturen” im Universum, grosse Galaxien z.B. formen sich dann über Verschmelzungen innerhalb der Filamente und Knotenpunkten.

    …kinetische Energie nimmt zu. . Die muss doch zum Verklumpen irgendwie weg

    Warum? Die ganzen Strukturen müssen nicht fixiert sein, sondern dürfen sich bewegen. Wir, also die Milchstrasse, bewegen uns mit einer gewissen Geschwindigkeit zum Lokalen Attraktor hin (https://de.wikipedia.org/wiki/Gro%C3%9Fer_Attraktor). Gleichzeitig zieht aber das Gravitationspotential der Milchstrasse Zwerggalaxien an.

  109. #109 Uli Schoppe
    3. Juni 2018

    Liegt jetzt vielleicht daran das ich mir Teilchen vorstelle? Da wird doch für Bewegungsänderung immer irgendwo Energie abgegeben / aufgenommen. Da kommt immer etwas Detektierbares herum. Wie machen DM Teilchen das?
    Ich bin wie gesagt vielleicht auf einem Holzweg..

  110. […] sondern nur Radialgeschwindigkeiten vorliegen – dass man auch damit arbeiten kann, haben wir bei NGC-1052-DF2 gesehen. Macht zusammen 143 Kugelsternhaufen, was eine fantastische Datenbasis ist (Watkins et al. […]

  111. #111 Till
    26. Juni 2018

    Klasse, nicht nur der Artikel war superinteressant sondern auch die Kommentare waren ein echtes Highlight. Vielen Dank an Alderamin, Oliver, Captain und die anderen Kommentatoren für diese fundierte und interessante Diskussion. Danke auch an “Nicht von Bedeutung” für das beisteuern des komischen Elementes (das nächste Mal bitte nur etwas kürzer fassen).

  112. […] Artikel NGC 1052-DF2 – Leuchtende Aussichten für Dunkle Materie hatte ich über eine Arbeit berichtet, in der Pieter van Dokkum behauptete, eine Galaxie gefunden […]

  113. […] Pieter van Dokkums Behauptung, eine Galaxie (fast) ohne Dunkle Materie gefunden zu haben – hier wurde darüber berichtet, und es gab bereits Kritik von anderer Seite an den Ergebnissen. Die […]

  114. #114 Alderamin
    12. September 2018

    @alle

    Wie ich eben erfahren habe, darf der im vorherigen Kommentar verlinkte Artikel erst heute um 19:00 Uhr veröffentlicht werden; die ursprüngliche Information lautete auf 17:00 Uhr. Daher bitte ich noch um ein wenig Geduld.

  115. […] gutes Beispiel um dies zu veranschaulichen ist die Entdeckung einer Zwerggalaxie ohne Dunkle Materie, publiziert in Nature um eine Gruppe in Yale (van Dokkum et al. 2018). Dabei haben sie geschrieben, […]

  116. #116 asperitias
    Berlin
    8. April 2019

    Vor einer Weile hat meine kleine Tochter (10), die sich extrem für Mathematik und Physik interessiert mir folgende Frage gestellt: Wenn es die dunkle Materie nicht gibt, kann es nicht sein, dass ein Parallel-Universum das andere anzieht und wir denken es ist unsichtbare Materie? Ich kann dazu nichts finden und konnte ihr auch nicht erklären, warum das nicht der Fall sein kann. Vor Jahren hat die NASA mal was bezüglich der dunklen Strömung gepostet, aber das entkräftet es auch nicht und ich bin was Physik angeht nur ein interessierter Laie, der zwar viel mit Mathe zu tun hat aber eher in Form von Spieltheorie oder Euler. Kann mir bitte jemand einen Link oder so etwas posten, wo ich mehr darüber raus finden kann. Ich würde es ihr schon gern erklären, nachdem ich sie mit meinem “Mathespleen” angesteckt habe. Mir würde auch eine PN an meine Email-Adresse (at)yahoo.de reichen. Wäre echt freundlich. Danke.

  117. #117 Captain E.
    8. April 2019

    @asperitias:

    Hm, das Problem ist nur, dass man mit Paralleluniversen alles und gar nichts erklären kann. Zumindest, solange wir keine Möglichkeit finden, die Hypothese zu testen, dass es derartige Paralleluniversen überhaupt gibt und was so in ihnen abgeht.

    Da dürfte die Möglichkeit, dass es da draußen irgendwelche völlig unbekannte und so gut wie nicht nachweisbare Teilchen gibt, erheblich wahrscheinlicher sein.

    Und wie bereits des öfteren erwähnt: Teilchen, die weder per elektromagnetischer Kraft noch per starker Kernkraft mit irgendetwas anderem interagieren, sondern höchstens mittels Gravitation und schwacher Kernkraft, wurden bereits nach Beobachtungen des Beta-Zerfalls postuliert – und später dann auch nachgewiesen:Neutrinos. Gerade heute habe ich noch einen Artikel über eine junge deutsche Wissenschaftlerin gelesen, die in Sachen Neutrinos am Südpol überwintert hat. Unglücklicherweise passt nicht alles, was wir für die Dunkle Materie benötigen. Sie sind zu energiereich/schnell und zu leicht/wenige. Der Rest passt aber.